Mandible 01-22 Flashcards

1
Q

A 25-year-old man was assaulted 72 hours ago and presents with a noncomminuted, minimally displaced right angle fracture of the mandible. The plastic surgeon plans to perform open reduction and internal fixation. Which of the following factors is most likely to place the patient at greatest risk for a postoperative complication?

A) Perioperative ampicillin-sulbactam administration
B) Placement of two mini-plates
C) Postoperative maxillomandibular fixation
D) Sex of patient
E) Time to operative intervention

A

The correct response is Option B.

In multiple studies, the use of a single plate along either the oblique ridge or the lateral cortex demonstrates the lowest incidence of complications for this type of fracture. Specifically, placement of two mini-plates has been associated with an increased risk for postoperative complication. Postoperative maxillomandibular fixation has not been shown to influence the incidence of complications. Furthermore, time to operative intervention and the sex of the patient have not been shown to increase the complication rate. Factors such as smoking and the number of fractures have been associated with an increased risk for complication with mandible fractures. Finally, perioperative ampicillin-sulbactam administration has been shown to decrease the risk for complications in this patient population.

How well did you know this?
1
Not at all
2
3
4
5
Perfectly
2
Q

A 26-year-old man presents to the emergency department because of jaw pain and malocclusion. He has normal dentition. CT scan shows an isolated, noncomminuted left-sided angle fracture of the mandible. Which of the following treatment methods is likely to have the lowest complication rate?

A) Closed reduction with arch bars
B) Open reduction through an intraoral approach with one miniplate
C) Open reduction through an intraoral approach with two miniplates
D) Open reduction through a submandibular incision with a 2.7-mm reconstructive plate

A

The correct response is Option B.

Treatment options for mandibular fractures are varied and include closed reduction and external fixation with intermaxillary fixation, as well as open reduction and internal fixation through either intraoral or extraoral approaches. Internal fixation strategies include non-rigid fixation with wire osteosynthesis, internal fixation with single or multiple miniplates, compression plate fixation, and locking reconstruction plates.

Intermaxillary fixation represents the historic standard for treating mandible fractures. Closed treatment of fractures with interdental fixation alone or combined with non-rigid internal fixation does not stabilize movement of the proximal segment in unstable and/or unfavorable angle fractures and results in an approximately 17% risk for infection or malunion.

Comparative studies and recent meta-analyses have demonstrated the lowest complication rates when treating isolated, noncomminuted mandibular angle fractures via open reduction and internal fixation with a single noncompression miniplate.

The addition of a second plate results in increased soft tissue and infection complications, likely because of the increased need for periosteal stripping and devascularization of the fractured segments.

Open reduction and fixation through an extraoral approach with a locking reconstruction plate results in slightly higher, but comparable, risk for complication. This treatment mandates a visible scar and carries increased risk for facial nerve injury. This treatment is indicated in comminuted fractures, fractures with loss of bone stock, and atrophic mandibles, but it is unnecessary in a patient with intact dentition and a noncomminuted, isolated fracture.

How well did you know this?
1
Not at all
2
3
4
5
Perfectly
3
Q

A 19-year-old man presents to the emergency department with malocclusion sustained during an assault. Examination shows isolated mandibular subcondylar fracture. A photograph is shown. Which of the following is the most likely location of the mandibular fracture?

A) Left
B) Right
C) Bilateral
D) Not possible to determine with information provided

A

The correct response is Option A.

Posterior mandibular fractures, such as those of the subcondylar subunit, cause foreshortening of the vertical height of the mandible and early contact of the molar teeth on the ipsilateral side of the fracture. This results in an open bite on the anterior contralateral side of the fracture as demonstrated in the picture (right open bite).

A displaced right subcondylar fracture usually presents with a left open bite.

The malocclusion pattern from bilateral subcondylar fractures is more difficult to predict. If displaced, they may present with an anterior open bite, caused by early contact of the molar teeth on both sides of the face.

How well did you know this?
1
Not at all
2
3
4
5
Perfectly
4
Q

A man presents to the emergency room with a complicated mandibular third molar infection. He has limited range of motion of the jaw (trismus) and point tenderness above the zygomatic arch. The infection is most likely in which of the following anatomic spaces?

A) Buccal
B) Prevertebral
C) Pterygomandibular
D) Submasseteric
E) Temporal

A

The correct response is Option E.

There are four separate compartments that comprise the masticator space, including masseteric or submasseteric space, pterygomandibular space, deep temporal space, and superficial temporal space. The submasseteric space is bordered by the masseter muscle and ascending ramus of the mandible. The pterygomandibular space is formed by the medial pterygoid muscle and ascending ramus. The superficial temporal space is formed by the temporalis fascia and temporalis muscle. The deep temporal space is formed by the temporalis muscle and calvarium. These four spaces function as “subspaces” of the masticator space, but they can all become involved rapidly once one compartment is affected. The submasseteric and superficial temporal spaces are separated by the zygomatic arch. The pterygomandibular and deep temporal spaces are separated by the lateral pterygoid muscle.

An infection in the prevertebral space would not present with the signs and symptoms of tenderness above the zygomatic arch, and would be more likely to present with neck rigidity or dysphagia.

How well did you know this?
1
Not at all
2
3
4
5
Perfectly
5
Q

An 11-year-old boy presents to the office after sustaining injury during a fall from his scooter. Orthopantomogram panoramic x-ray study shows displaced fracture of the body of the mandible on the left side, in the line of the first premolar. On clinical examination, the permanent lateral incisor appears longer than the lateral incisor on the right side. The permanent second molars have not yet erupted. Which of the following statements regarding management of this patient’s injury is most accurate?

A) Circummandibular wiring is contraindicated in pediatric mandible fracture because the wires can damage tooth buds
B) Maxillomandibular fixation with arch bars is contraindicated in pediatric patients during primary and mixed dentition
C) Plate fixation of mandibular fractures during mixed dentition should never be performed
D) A tooth in the fracture line is indicated for removal
E) A tooth that has been luxated from its socket is indicated for removal

A

The correct response is Option E.

The longer-appearing lateral incisor is an example of an extrusive luxation (versus an intrusive luxation where the tooth is impacted).

A recent study by Naran et al reviewed 154 mandibular fractures in children in primary or mixed dentition and found no disturbances to permanent dentition with the use of archbars. Therefore, it is not contraindicated.

The Arbeitsgemeinschaft für Osteosynthesefragen, or AO, foundation has listed indications for tooth removal in the setting of facial fractures, and these include: fractured tooth roots, a tooth that has been luxated from its socket, a tooth that is interfering with reduction of the fracture, advanced dental caries that would carry risk for infection/abscesses, advanced periodontal disease, teeth with pre-existing abnormalities.

A tooth in the line of fracture alone is not an indication for removal (Kumar).

Circummandibular wiring can be used in pediatric mandible fractures as well as the edentulous mandible fracture.

Plate fixation during mixed dentition can be performed, depending on the location, after the permanent dentition has erupted. For example, plating of the symphseal region is possible after eruption of permanent incisors, and the parasymphseal region after eruption of permanent canines.

How well did you know this?
1
Not at all
2
3
4
5
Perfectly
6
Q

A 76-year-old man sustains a right mandibular body fracture after a mechanical fall. He states that he lives independently and is active. On examination, the patient is noted to be edentulous. A CT scan demonstrates a comminuted fracture mesial to the angle without evidence of any other injuries. Which of the following is the most appropriate treatment of this deformity?

A) Closed reduction and external fixation
B) Intra-oral incision and miniplate fixation along external oblique ridge
C) Maxillomandibular fixation
D) Mechanical soft diet for four weeks
E) Submandibular incision and reconstruction bar fixation

A

The correct response is Option E.

In edentulous patients, mandibular atrophy can make it difficult to achieve appropriate reduction. Mandibular body fractures are common in these patients following blunt trauma, such as a fall. Regarding the management of a mandible fracture in an edentulous man, open reduction and internal fixation is necessary to provide long-term stability and an accurate restoration of previous anatomy, possibly due to poor osteogenic capacity and ability to load-bear of an atrophic mandible.

Intra-oral and extra-oral incisions are both acceptable exposure methods. Similarly, miniplates or reconstruction bars can both be utilized, although some prefer the added stability of a reconstruction bar. Submandibular incision and reconstruction bar fixation is the most correct initial management, as it involves open reduction and internal fixation of the fracture.

Mechanical soft diet is not appropriate treatment for a comminuted mandibular body fracture and will result in malunion or nonunion. Similarly, the use of maxillomandibular fixation, with or without dentures, does not accurately reduce the fracture in an edentulous mandible with intrinsic loss of osteogenic potential and can lead to higher complications, such as pneumonia, in elderly patients. Miniplate fixation along the external oblique ridge is a treatment for noncomminuted angle fractures, but is not the appropriate treatment for a comminuted body fracture.

External fixation is usually reserved for injuries with significant soft tissue deficit. It does not provide accurate anatomic alignment.

How well did you know this?
1
Not at all
2
3
4
5
Perfectly
7
Q

A 22-year-old woman is brought to the emergency department after being hit by a car where she was intubated at the scene. A maxillofacial CT scan performed on admission shows a non-displaced right mandibular parasymphaseal fracture. She is extubated later that day after being cleared by trauma. Upon reevaluation the next morning, significant malocclusion with a step-off between the right mandibular cuspid and lateral incisor is noted. She has a right-sided open bite. Which of the following mechanisms most likely explains the change in physical examination findings after extubation?

A) Delayed disruption of the periodontal ligament
B) Differential pull of muscles on the mandible
C) Dissipation of post-traumatic edema
D) Fibrinolysis of the fracture hematoma
E) Refracture of the parasymphysis

A

The correct response is Option B.

This patient has a right mandibular parasymphyseal fracture that is in an unfavorable orientation. The right temporalis and pterygomasseteric sling will work naturally to close the mandible while the floor of mouth musculature, including the mylohyoid, work to open the mandible at the level of the symphysis. Because of the orientation of the fracture, these forces distract the fracture, causing it to become more displaced. When this patient was first examined, the paralytic agent she received when she was intubated at the scene inhibited those distraction forces and kept her in appropriate occlusion. However, upon extubation, she fell out of occlusion as those muscle groups separated the fracture. While hematoma fibrinolysis and edema reduction occur after mandible fractures, they are not likely to result in displacement of the fracture. The periodontal ligament has no role in fracture displacement.

How well did you know this?
1
Not at all
2
3
4
5
Perfectly
8
Q

A 25-year-old man presents with a diagnosis of bruxism and pain. Which of the following drugs is the most appropriate for treatment?

A) Amitriptyline
B) Botulinum toxin type A
C) Clonidine
D) Escitalopram
E) Fluoxetine

A

The correct response is Option B.

Botulinum toxin A has shown efficacy in the treatment of pain symptoms from bruxism and, thus, its effects are not limited to improving masseter hypertrophy. It has shown equal efficacy when compared with occlusal splints.

Botulinum toxin A has been shown to be more effective than placebo in decreasing pain from bruxism by a patient report using the visual analogue scale (VAS). However, other pharmacotherapies, such as clonidine and amitriptyline, have demonstrated no improvement in pain relief over placebo.

Bruxism is a reported side effect of selective serotonin reuptake inhibitors such as fluoxetine and escitalopram.

How well did you know this?
1
Not at all
2
3
4
5
Perfectly
9
Q

A 35-year-old woman is unhappy with the appearance of her “square face,” especially at the bottom jaw “near the corners,” and she wishes to have a smoother mandible contour permanently. Which of the following is the most appropriate treatment?

A) Alloplastic implant
B) Autologous fat grafting
C) Hyaluronic acid dermal filler
D) Mandible angle and body contouring
E) Suction lipectomy

A

The correct response is Option D.

Mandible contouring surgery, frequently called mandible angle reduction, is a bony procedure to decrease the angular contours in a “square face” or bottom jaw with “sharp corners.” The term “mandible angle” reduction is a misnomer, as usually both the mandible angle and the mandible body need to be gracefully contoured or resected to result in an aesthetically pleasing, rounder face.

Although fillers, fat grafting, and implants are used in the face, they are not usually used for mandible contouring. Suction lipectomy for a patient with a square jaw would not be successful.

How well did you know this?
1
Not at all
2
3
4
5
Perfectly
10
Q

A 40-year-old man presents to the emergency department with a fracture of the mandible. There is concern that he also has a cervical spine injury. Which of the following is an independent risk factor for concomitant cervical spine injury in this patient?

A) Alcohol use as the cause of the injury
B) Chest injury
C) Facial laceration
D) Personal assault as the cause of the injury
E) Symphyseal fracture

A

The correct response is Option B.

Concomitant cervical spine injuries (CSI) can occur with mandible fractures. CSI with mandible fractures have been reported in 4.4 to 10% of cases. Recently, independent risk factors for concomitant CSI were investigated. Results show that chest injury and ramus-condyle unit fractures are independent risk factors for concomitant CSI. Patients with mandible fractures and associated mid face/head fractures have increased risk for CSI. Additionally, the risk for concomitant CSI is increased if the cause of the trauma was non-craniomaxillofacial injury or motor vehicle collision. Although other items listed are risk factors associated with mandible trauma, they have not been found to be independent risk factors for concomitant CSI.

How well did you know this?
1
Not at all
2
3
4
5
Perfectly
11
Q

A 31-year-old man presents with a posterior fracture to the body of the mandible involving the alveolus of the first molar sustained during an assault. Open reduction and internal fixation of the fracture is performed. Intraoperatively, the position of the mandibular first molar in the fracture prevents an adequate reduction, and it must be extracted. Removal of how many intact tooth roots is most likely to indicate complete extraction of the mandibular molar in this patient?

A) One
B) Two
C) Three
D) Four

A

The correct response is Option B.

Anatomy of the mandibular first molar is relatively consistent in that the vast majority will have two roots. Knowledge of the number of roots is important in the setting of extraction to ensure complete removal. If either the injury or the reduction of the mandible fracture had caused a fracture of the tooth root itself, then complete removal of the fractured root would involve either exploration at the time of open reduction and internal fixation or postoperative referral to an oral surgeon. A retained tooth root would place the patient at high risk for abscess formation that could require additional treatment. Incisors, canines, mandibular premolars, and maxillary second premolars usually have one root. Maxillary first premolars and mandibular molars usually have two roots. Maxillary molars usually have three roots.

How well did you know this?
1
Not at all
2
3
4
5
Perfectly
12
Q

A 22-year-old man is evaluated because of a painless, firm, unilateral enlarging mass of the body of the mandible. He denies trauma to the area and he has excellent oral hygiene. His dentist performed fine needle biopsy that showed multinucleated giant cells. CT scan shows a radiolucent bone lesion with an expanded cortex. Which of the following is the most appropriate next step in management?

A) Incisional biopsy
B) Partial mandibulectomy with free margins
C) Radiation therapy
D) Resection and curettage
E) Sclerotherapy

A

The correct response is Option D.

This patient has an aneurysmal bone cyst (ABC). These lesions may be related to giant cell granulomas. These lesions are most common in the long bones with 1.9% of them being reported in the mandible.

The correct answer is resection and curettage. In a study of 120 ABC cases, resection and curettage was reported to have a 91.8% success rate (recurrence occurred in 11 out of 120 cases). Incomplete resection is hypothesized to be a cause of recurrence. Recurrence was not related to histopathologic parameters.

Pathologically, these lesions are a pseudocyst comprised of multinucleated giant cells, woven trabecular bone with caverns, and sinusoids lacking endothelium. Recurrence can be treated with repeat excision curettage, open packing, or block resection. These lesions are quite vascular and typically bleed until resected, so expeditious removal is recommended (transfusion with packed red blood cells has been reported in the literature).

Incisional biopsy would be diagnostic but is not recommended for these lesions given their vascularity and the surgeon’s inability to control the bleeding.

Sclerotherapy is recommended for vascular malformations such as arteriovenous malformations, venous malformations, or lymphatic malformations. This lesion would not be responsive to this type of therapy.

Partial mandibulectomy with free margins is the preferred treatment for lesions like ameloblastomas. Given the efficacy of excision and curettage, mandibulectomy is not the recommended first line treatment for this diagnosis. It can be employed in recurrences (although as stated above, less aggressive interventions are typically employed first).

How well did you know this?
1
Not at all
2
3
4
5
Perfectly
13
Q

A 35-year-old woman has minimally displaced bilateral subcondylar fractures of the mandible without loss of posterior vertical height, but she reports subjective malocclusion. Which of the following is the most appropriate management of this patient?

A) Advise the patient to eat a soft diet for 6 weeks
B) Application of a gunning splint for 8 weeks
C) Maxillomandibular fixation (MMF) for 4 to 6 weeks
D) MMF for 1 week
E) MMF for 8 to 10 weeks

A

The correct response is Option C.

Closed reduction has historically been the standard treatment option for subcondylar fractures of the mandible. Its widespread use is attributed to the idea that closed reduction results in fewer complications with similar functional and aesthetic outcomes compared with open reduction and internal fixation (ORIF). For instance, complications such as facial nerve damage and excessive scarring are significantly decreased due to the noninvasive nature of this approach. However, as highlighted by the ongoing debate, a consensus regarding outcomes between open and closed reduction is not evident in the literature. In short, some studies conclude that both approaches produce roughly similar results, while others have associated an array of unfavorable outcomes with closed reduction. These include facial asymmetry, deviation upon mouth opening, skeletal malocclusion, and chronic pain of the temporomandibular joint (TMJ). The fact that many of these parameters lack standardization in time course of treatment further obscures the debate. Larger studies with consistent parameters are needed to reassess outcomes with the surgical techniques and technology present today. However, it is unlikely that a large enough trial will deliver granular evidence to conclusively quell this debate.

Another controversial point regarding closed reduction is the length of time a patient should spend in maxillomandibular fixation (MMF). Many surgeons choose to apply fixation for a very short period (ie, 2 weeks) to avoid ankylosis of the TMJ secondary to forced immobilization during MMF. While the etiology of ankylosis is not completely understood, it is hypothesized that trauma leading to intracapsular hematoma results in fibrosis and excessive bone formation, ultimately causing hypomobility of the affected side.

Given the current hypothesis, ankylosis of the TMJ is likely a manifestation of direct injury within the joint capsule or condylar head itself. It is imperative to point out that as a result, there should be a decreased risk for ankylosis in subcondylar fractures compared with fractures of the condylar head. Therefore, the position of the fracture line relative to the joint capsule should be closely examined, and a longer period of MMF should be employed if there is no involvement of the condylar head, disc, or capsule. A longer period of MMF results in better union of the fractured segments with no increase in the incidence of ankylosis. In a nondisplaced fracture or minimally displaced fracture with a functional occlusion, 4 to 6 weeks of MMF followed by 2 to 3 weeks of guiding elastics is recommended. The same treatment applies in the case of nondisplaced bilateral fractures. However, this scenario is less common because the force parameters to cause the bilateral fractures are often greater and tend to displace the fracture fragments significantly, necessitating ORIF.

How well did you know this?
1
Not at all
2
3
4
5
Perfectly
14
Q

A 12-year-old boy is brought to the office for evaluation of an obvious anterior open bite sustained when he fell from his bicycle. Imaging shows a displaced fracture of the right mandibular condyle with intra-articular extension. Which of the following is the most appropriate treatment?

A) Application of an external fixator
B) Delayed sagittal split osteotomy
C) Maxillomandibular fixation with arch bars
D) Open reduction and internal fixation
E) Soft diet and observation

A

The correct response is Option C.

Condylar fractures in children can predispose to facial growth disturbance and temporomandibular joint dysfunction. A condylar fracture with an associated parasymphyseal fracture and an open bite should be treated with arch bars and a period of intermaxillary fixation in a 12-year-old, if possible. Open reduction and plating of the mandible is generally avoided in this age group to avoid injury to tooth buds. It has been demonstrated that arch bars can be used safely and effectively for the injury pattern described during the period of mixed dentition. Delayed sagital split osteotomy is not indicated in a patient with a normal premorbid occlusion.

How well did you know this?
1
Not at all
2
3
4
5
Perfectly
15
Q

A 22-year-old man is scheduled to undergo surgery to correct a displaced symphyseal fracture of the mandible using a vestibular approach. To avoid injury to the mental nerve, the mental foramen is best identified in which of the following locations?

A) Anterior to the first premolar
B) Inferior to the lateral incisor
C) Inferior to the lower canine tooth
D) Inferior to the second molar
E) Inferior to the second premolar

A

The correct response is Option E.

The vestibular approach to the mandible is useful for a number of mandibular procedures. The most important neurovascular structure of significance in the region of the symphysis is the mental neurovascular bundle. The mental nerve is the terminal branch of the inferior alveolar nerve. It gives sensation to the skin and mucosa of the lower lip, skin of the chin, and facial gingiva of the anterior teeth. It is important to preserve this nerve during the surgical dissection in this approach. The mental nerve exits the mental foramen usually inferior to or slightly anterior to the second premolar tooth.

How well did you know this?
1
Not at all
2
3
4
5
Perfectly
16
Q

Which of the following provides the most significant blood supply to the mandibular condylar head?

A) Articular disk
B) Capsule of the temporomandibular joint
C) Deep branch of the superficial temporal artery
D) Facial artery
E) Medullary branch of the inferior alveolar artery

A

The correct response is Option E.

Concerning the condyle itself, its blood supply is mostly derived from three sources. A branch of the inferior alveolar artery courses upward through the neck of the condylar process, where it anastomoses liberally with vessels from the attached musculature. Another major contributor to the condyle and its articular surface derives from the temporomandibular joint (TMJ) capsule, with its lush vascular plexus. A large supply of blood also comes from branches of the lateral pterygoid muscle through its attachment at the pterygoid fovea. Of these three sources, the medullary blood supply from a branch of the inferior alveolar artery was found to be the most important source in monkeys and, presumably, in man. Fracture of the subcondylar or neck region of the condylar process could therefore disrupt the main vascular supply to the condyle.

There is another ramification of the loss of medullary blood supply from fracture of the condylar process. Surgical access to the condylar process in order to perform open reduction and internal fixation requires exposure and dissection of some of the soft tissues from the condylar process to permit manipulation and attachment of fixation devices. Therefore, surgery further diminishes the blood supply to a segment of bone that has already been severely compromised. If maintenance of blood supply to the condyle is important, the best choice is a surgical approach that can minimize the amount of soft-tissue stripping from the fractured condylar process and maintain, as much as possible, the attachment of the TMJ capsule and the lateral pterygoid muscle. Thus, if the preauricular approach is chosen, one should not enter the capsule of the joint as one might for an intra-articular surgery. Doing so can disrupt the already compromised blood supply to the condyle.

How well did you know this?
1
Not at all
2
3
4
5
Perfectly
17
Q

A 27-year-old woman is evaluated for jaw pain and new-onset malocclusion with anterior open bite, resulting from a fall from a ladder 6 hours ago. X-ray studies show comminuted bilateral intracapsular mandible fractures. Which of the following is the most appropriate treatment?

A) Application of an external fixation
B) Bilateral temporomandibular joint replacements
C) Maxillomandibular fixation for 2 weeks
D) Open reduction and internal fixation
E) Observation with soft diet restriction

A

The correct response is Option C.

Open reduction and internal fixation of isolated intracapsular fractures of the mandible condyles is not recommended—the fragments are too small to provide reliable hardware fixation. Recommended treatment is a course of closed reduction and maxillomandibular fixation. A period of immobilization with restoration of preinjury occlusion, rather than observation alone, would help relieve pain and restore preinjury bone and soft-tissue relationships. External fixation would offer no benefit in the presence of normal dentition and no other complex injury pattern of the mandible, i.e., segmental loss with accompanying soft-tissue deficit. Replacement of TMJ in this patient is not indicated given her age and degree of injury.

How well did you know this?
1
Not at all
2
3
4
5
Perfectly
18
Q

A 25-year-old man with a fracture of the mandibular angle is evaluated for open reduction and internal fixation. Which of the following is the most appropriate location for placement of internal fixation when using the Champy technique in this patient?

A) External oblique ridge
B) Lingula
C) Mandibular notch
D) Mental tubercle
E) Mylohyoid line

A

The correct response is Option A.

This technique for internal fixation of mandibular angle fractures was first described by Michelet in 1972, and biomechanical studies by Champy et al were published in 1976. In his original paper, Champy advocated placement of an internal fixation miniplate along the external oblique line of the mandible. By contouring the plate to this line, rigid fixation is achieved in two planes, taking advantage of the force vectors along the angle of the mandible to help compress the fracture site.

Mylohyoid line is not appropriate because the mylohyoid line is located along the lingual surface of the mandibular body and is not used routinely as a location for plate fixation for mandibular fractures.

Lingula is not appropriate because the lingula is located along the lingual surface of the mandibular ramus, and is the location where the mandibular nerve (V3) enters the mandible. Plate fixation should be avoided in this area because of the difficulty of exposure and the risk for damaging the mandibular nerve.

Mental tubercle is not appropriate because the mental tubercle is not an appropriate place to apply internal fixation when utilizing the Champy technique.

Mandibular notch is not appropriate because it is located between the mandibular condyle and the coronoid process and is not an appropriate place to apply internal fixation when utilizing the Champy technique.

How well did you know this?
1
Not at all
2
3
4
5
Perfectly
19
Q

A 16-year-old girl is brought to the emergency department after sustaining fractures to the mandible in a rollover motor vehicle collision. She is intubated during the primary survey because of her critical airway. A CT scan is shown. Which of the following is the most appropriate treatment of this fracture pattern?

A) Maxillomandibular fixation only
B) Observation and soft diet
C) Open reduction and internal fixation of both the right body and the left angle fractures
D) Open reduction and internal fixation of the right body fracture only
E) Secondary mandible reconstruction after 6 weeks

A

The correct response is Option C.

This CT scan shows right–body and left–angle fractures, both with clinically significant displacement. Modern facial fracture management dictates early primary reduction and repair; therefore observation and soft diet without surgery is incorrect, as is avoiding disturbance of the mandible and allowing 6 weeks of bone healing. Since she has reached the age of skeletal maturity permanent titanium hardware is an accepted modality of treatment; therefore it is incorrect to avoid hardware and rely on non-reduced maxillomandibular fixation. In addition, an angle fracture cannot typically be treated with maxillomandibular fixation alone, but requires a Champy plate or more rigid fixation. Open reduction and internal fixation of the right–body fracture, without repairing the left angle is incorrect, as Champy lines do not preclude the need for fixation. Open reduction and internal fixation of both the right body and the left angle fractures is correct, as both fractures need to be reduced, and then fixated, for the best chance of restoring occlusion.

How well did you know this?
1
Not at all
2
3
4
5
Perfectly
20
Q

A 27-year-old man is admitted to the emergency department after being injured in an altercation. Physical examination shows objective malocclusion with a left-sided crossbite and right-sided open bite. A CT scan is shown. Which of the following muscles is most likely responsible for these radiographic and physical examination findings?

A) Genioglossus
B) Lateral pterygoid
C) Masseter
D) Medial pterygoid
E) Mylohyoid

A

The correct response is Option B.

The patient described has typical radiographic and physical exam findings of a right subcondylar mandibular fracture. Anteromedial displacement of the condylar segment out of the glenoid fossa occurs secondary to pull from the lateral pterygoid muscle, which normally functions in anterior translation of the condyle across the articular eminence of the temporal bone during wide mouth opening. This leads to loss of height of the mandibular ramus and a premature occlusion on the fracture side. This causes the typical findings of condylar/subcondylar fractures: ipsilateral crossbite and contralateral open bite. The masseter and medial pterygoid form the pterygomasseteric sling, which attaches from the skull base and zygoma to the inferior mandibular border, and is responsible for fracture displacement after angle and body fractures. The mylohyoid and genioglossus muscles run along the floor of the mouth and can contribute to fracture displacement in the body and parasymphyseal region.

How well did you know this?
1
Not at all
2
3
4
5
Perfectly
21
Q

An 18-year-old man is evaluated for a mandibular fracture sustained in a motor vehicle collision. CT scan shows displaced left parasymphyseal and right mandibular body fractures. Resorbable plate fixation is planned. Compared with titanium implants, the use of resorbable fixation to treat this fracture is associated with which of the following?

A) Decreased postoperative pain
B) Decreased risk for malunion
C) Increased overall cost
D) Increased risk for infection
E) Lower hardware profile

A

The correct response is Option C.

Bioresorbable fixation is now widely employed in craniomaxillofacial surgery. While this technology has theoretical advantages in the treatment of craniosynostosis (e.g., obviates concerns of intracranial implant migration), the benefits of orthognathic surgery and the management of facial trauma are dubious. For mandibular fractures, several studies have found no statistical differences in overall or specific complication rates, including the need for plate removal, postoperative infection, malreduction/malocclusion, postoperative pain, or loss of fixation. Rigorous comparisons are lacking, but a recent comprehensive review of the reported studies demonstrated a trend toward increased complications using resorbable fixation to treat facial fractures. The only consistent difference is the cost of the implants, which is considerably higher for the resorbable systems.

How well did you know this?
1
Not at all
2
3
4
5
Perfectly
22
Q

A 63-year-old woman is admitted to the hospital with a fracture to the left mandibular angle that she sustained in a motor vehicle collision. Open reduction and internal fixation is planned via a submandibular (Risdon) incision. The region between which of the following planes of dissection is most appropriate for approaching the fracture and avoiding injury to the marginal mandibular branch of the facial nerve?

A) Carotid sheath and the posterior belly of the digastric muscle
B) Platysma and the superficial (investing) layer of the deep cervical fascia
C) Skin and the platysma
D) Submandibular gland and the carotid sheath
E) Superficial (investing) layer of the deep cervical fascia and the submandibular gland

A

The correct response is Option E.

The submandibular approach to the mandibular body/ramus is an important surgical approach to address a number of facial fractures. An understanding of the anatomy of the submandibular region is crucial to performing this approach with minimal complications. The marginal mandibular nerve leaves branches off the facial nerve trunk during its intraparotid course. One to three branches usually exit the inferior border of the parotid gland before changing course to travel along the mandibular border toward the symphysis. In almost all cases, the marginal mandibular nerve travels superficial to or within the investing layer of the deep cervical fascia as it courses anteriorly. In several cadaveric studies, the nerve consistently coursed within 1 cm below the mandibular border posterior to the facial vessels, and above the mandibular border anterior to the facial vessels. The submandibular approach to facial fractures involves incising the skin 2 cm below the mandibular border, dividing the platysma and dissecting deep to the investing fascia, just above the submandibular gland. Once in this plane, the course of dissection is carried superiorly toward the mandibular border.

How well did you know this?
1
Not at all
2
3
4
5
Perfectly
23
Q

A 50-year-old man is brought to the emergency department after a high-speed motor vehicle collision. CT scan is obtained and shows a highly comminuted fracture of the mandible involving the symphysis, body, and angle on the left. Which of the following types of vessels supplies perfusion to these mandibular segments?

A) Diaphyseal
B) Epiphyseal
C) Galeal
D) Metaphyseal
E) Periosteal

A

The correct response is Option E.

In severely comminuted fractures of the mandible, the viability of the bony fragments depends on the blood supply from the periosteum and periosteal vessels. Significant periosteal stripping during an open repair would place these patients at risk for necrosis and bony resorption. This is the premise behind closed reduction of these injuries when indicated.

There are two major types of bone: tubular (long bone) and flat bones (primarily of the facial skeleton). Tubular bones include the long bones of the extremities, clavicles, hands, and feet, and are composed of a diaphysis, paired metaphyses, and epiphyses. Tubular bones have a dual blood supply. The predominant supply is the nutrient diaphyseal arteries that often enter the middle third of the diaphysis and bifurcate upon entering the medullary canal. At the distal aspect, the smaller metaphyseal and epiphyseal arteries generally arise from arteries that supply the joint and anastomose with the diaphyseal arteries. This less robust blood supply at the joints explains why fractures in this location can lead to growth retardation. The other major blood supply is the periosteal vascular plexus; this plexus relies on connections with overlying skeletal muscle.

Flat bones of the facial skeleton, as well as the scapulae, sternum, and ribs, do not contain a diaphysis, metaphyses, or epiphyses. They contain a dual blood supply with nutrient arteries (generally based off of the maxillary artery or middle meningeal artery for the calvaria) and the periosteal vessels and vascular plexus. Although the vascular connections with the periosteum are poorly developed, vascularized calvarial grafts could be based on this system if the galea and vessels were adherent to the graft. There is no galea on the mandible and, therefore, no galeal vessels.

How well did you know this?
1
Not at all
2
3
4
5
Perfectly
24
Q

A 16-year-old boy is evaluated for multiple fractures to the mandible after crashing his ATV into a tree. Which of the following modalities best takes advantage of load-bearing osteosynthesis in the management of this patient’s fractures?

A) Bicortical border plate for a mandibular parasymphyseal fracture
B) Champy plate fixation for a mandibular angle fracture
C) Lag screw fixation for a displaced fracture of the mandibular symphysis
D) Locking reconstruction plate for a comminuted mandibular body fracture
E) Maxillomandibular fixation for a minimally displaced mandibular subcondylar fracture

A

The correct response is Option D.

Load-bearing osteosynthesis differs from load-sharing osteosynthesis in that the fracture plate assumes all of the load of the given bone rather than distributing the load among the plate and the bone. Examples of load-sharing osteosynthesis include lag screw fixation, maxillomandibular fixation, nonlocking mandibular border plate, and monocortical miniplate (Champy principle) fixation. An example of load-bearing fixation is a mandibular locking reconstruction plate for a comminuted fracture.

How well did you know this?
1
Not at all
2
3
4
5
Perfectly
25
Q

A 45-year-old woman with a history of metastatic breast cancer previously treated with zoledronate is evaluated because of an area of exposed necrotic mandibular bone measuring 1 × 1 cm. She has no pain, and there is no clinical evidence of infection. Panoramic x-ray study (Panorex) shows no evidence of fracture. Which of the following is the most appropriate next step in management?

A) Aggressive debridement and prophylactic titanium plate placement
B) Antiseptic mouth rinses and observation
C) Dental extraction and intravenous antibiotics
D) Segmental mandibulectomy and osteocutaneous free flap reconstruction
E) Superficial debridement and oral antibiotics

A

The correct response is Option B.

This patient has osteonecrosis of the jaw secondary to bisphosphonate medication used to treat her bony cancer metastases. The mandible is more commonly affected than the maxilla. The appropriate next step in management is to begin antiseptic mouth rinses in addition to stressing good dental hygiene. Asymptomatic bone exposure can be followed for progression, and early cases appear to often resolve spontaneously when the bisphosphonates are discontinued.

Ruggiero et al. proposed a three-stage classification system and treatment algorithm for bisphosphonate-related osteonecrosis of the jaw based on clinical features. In Stage 1, there is exposed and necrotic bone that is otherwise asymptomatic, as in the patient described above. In Stage 2, there is exposed and necrotic bone with pain and clinical evidence of infection. Antiseptic mouth rinses, oral antibiotics, and superficial debridement are recommended for this stage. Stage 3 includes all the features of Stage 2 and one or more of the following: pathologic fracture, extraoral fistula, and osteolysis extending to the inferior mandibular border. Stage 3 patients require surgical debridement or resection in addition to antiseptic mouth rinses and oral antibiotics. Dental extraction of infected, unsalvageable teeth should be performed, but extractions may also result in further nonhealing wounds and bone exposure. Segmental mandibulectomy is usually followed by immediate reconstruction with osseous or osteocutaneous free flap reconstruction. The role for prophylactic titanium plating of the jaws to prevent pathologic fracture has not been studied.

How well did you know this?
1
Not at all
2
3
4
5
Perfectly
26
Q

In the pediatric population, which of the following mandibular fracture patterns most commonly results in future growth complications?

A) Displaced bilateral parasymphyseal fractures
B) Displaced mandibular body fractures
C) Nondisplaced bilateral condylar fractures
D) Nondisplaced bilateral parasymphyseal fractures
E) Nondisplaced mandibular symphyseal fractures

A

The correct response is Option C.

Nondisplaced mandibular symphyseal fracture is incorrect as the mandible growth centers are located in the condyle and posterior aspect of the ascending ramus of the mandible.

Displaced bilateral parasymphyseal fractures is incorrect.

Nondisplaced bilateral condyle fractures is correct because the growth centers are located in the mandible condyle regions and a significant amount of trauma resulting in displaced factors of the condyles has occurred.

Displaced mandible body fractures is incorrect.

How well did you know this?
1
Not at all
2
3
4
5
Perfectly
27
Q

A 6-year-old boy is brought to the emergency department following facial trauma from falling on his bicycle handlebars. Which of the following mandible fracture locations is most commonly associated with anterior open bite?

A) Angle
B) Body
C) Coronoid
D) Subcondylar/condylar
E) Symphysis/parasymphysis

A

The correct response is Option D.

Anterior open bite, also known as apertognathia, is vertical separation of the maxillary and mandibular anterior teeth. It is caused by premature contact of the posterior molars, most commonly following bilateral subcondylar mandible fracture. When present, a unilateral subcondylar/condylar fracture causes an open bite on the side opposite the fracture.

How well did you know this?
1
Not at all
2
3
4
5
Perfectly
28
Q

A 32-year-old man undergoes open reduction and internal fixation of a fracture in the left body of the mandible. A six-hole dynamic compression plate is chosen for the inferior mandibular border. The first screw is placed bicortically through a plate hole immediately adjacent to the line of fracture. To obtain maximum compression at the fracture line using the spherical gliding principle, the most appropriate next step is to drill for a second screw at which of the following locations (A–E)?

A

The correct response is Option A.

To obtain maximum compression at the fracture line using the spherical gliding principle, the most appropriate next step is to drill for a second screw eccentrically, away from the line of fracture, through a plate hole located across the fracture line.

Concentric or neutral drilling occurs in the center of a plate hole, while eccentric drilling occurs in the periphery (corner) of the plate hole (either away or closer to the line of fracture).

The figure above illustrates the spherical gliding principle in a dynamic compression plate. The special geometry of the plate hole—together with eccentric, away from the fracture line, placement of the screw that has a spherically shaped head—allows interfragmentary compression in an axial direction when the screw is driven fully into the plate hole. For appropriate fracture compression to occur, the head of the screw that was placed first must be well seated into the plate hole, stabilizing the plate against the underlying bony segment.

Drilling concentrically (neutrally) through a plate hole located across the fracture line would cause no axial movement of the underlying bone fragments upon tightening of the screw against the plate, generating no further compression at the fracture line.

Drilling eccentrically, closer to the line of fracture, through a plate hole located across the fracture line would increase the fracture gap upon tightening of the screw against the plate.

Placement of a second screw in a plate hole located on the same side of the first screw (in relation to the line of fracture) would have no effect over the fracture line. Also, after eccentric (away from the line of fracture) placement of one screw on each side of the fracture, the remaining screws should be placed concentrically.

How well did you know this?
1
Not at all
2
3
4
5
Perfectly
29
Q

A 62-year-old woman who underwent chemotherapy/radiation protocol for oropharyngeal cancer 10 years ago has onset of severe pain after a dental extraction. Subsequent CT scan shows a pathologic fracture of the mandibular angle. Which of the following is the most appropriate management?

A) Hyperbaric oxygen therapy
B) Long-term intravenous antibiotic therapy
C) Oncology consultation
D) Open reduction and internal fixation of the mandible
E) Resection and coverage with a fibular free flap

A

The correct response is Option E.

Over the past few decades, the use of chemotherapy/radiation as the primary curative treatment for oropharyngeal cancer has increased. In part, this has to do with cure rates and tissue preservation, but it is also due to the rise in human papillomavirus–positive oropharyngeal cancer. Not surprisingly, there has been a marked increase in osteoradionecrosis of the mandible, in particular. The most common cause of pathologic fracture after radiation therapy in the mandible is tooth extraction, usually the third molar, and a subsequent angle injury, as in this case. With a large, multi-decade experience in the use of osseous free flaps, especially the fibular flap, these cases are now routinely managed by resection of the affected bone and immediate reconstruction.

Hyperbaric oxygen, as a single modality for osteoradionecrosis, is at best controversial and would not cure a pathologic fracture.

Intravenous antibiotics can treat osteomyelitis, but in a case of osteoradionecrosis and a fracture, a short course of adjuvant antibiotics (along with appropriate surgery) would be sufficient, at best.

Open reduction and internal fixation of this fracture would not suffice either, because necrotic bone will not heal, even if put into juxtaposition.

Although it is important to assume that any pathology in cases like these are cancerous until proven otherwise, nevertheless, this scenario as described is very common and the constellation of events plus the imaging indicates that oncology’s role in this case would be limited at best.

How well did you know this?
1
Not at all
2
3
4
5
Perfectly
30
Q

A 25-year-old man comes to the office because of jaw pain after sustaining a punch to the face 2 days ago. Maxillofacial CT scan shows a displaced comminuted fracture of the left mandibular angle. Open reduction and internal fixation is planned. Which of the following methods of fixation is considered load-bearing osteosynthesis?

A) Champy plate (oblique ridge)
B) Compression plates
C) Lag screws
D) Locking reconstruction plate
E) Simple screws

A

The correct response is Option D.

Load-bearing osteosynthesis of the mandible may be accomplished with a reconstruction plate and locking screws.

There are two basic types of mandibular fracture fixation: load-bearing osteosynthesis and load-sharing osteosynthesis.

In load-bearing osteosynthesis, the plate (or external fixator) assumes all the forces of mandibular function at the fracture site. Common clinical indications include comminuted fractures, fractures with segmental defects, and those in the atrophic edentulous mandible.

In load-sharing osteosynthesis, stability at the fracture site is created by the frictional resistance between the end of the bone and the hardware used for fixation. This requires adequate bony buttressing across the fracture line. Lag screws, Champy plate (at the oblique ridge), compression plates, and simple screws are examples of load-sharing osteosynthesis, which may have different levels of force distribution between the hardware and the bone.

How well did you know this?
1
Not at all
2
3
4
5
Perfectly
31
Q

A trauma patient with an isolated mandible fracture, which of the following is the likelihood that this patient will have a concomitant cervical spine injury?

A) 1%
B) 5%
C) 15%
D) 25%
E) 50%

A

The correct response is Option B.

In a recent review of the National Trauma Data Bank of over 1.3 million trauma patients, an analysis of isolated facial fractures and neurologic injury was undertaken. An isolated mandible fracture had a 5.1% relationship with a cervical spine injury. These data showed a higher than previously published rate of cervical spine and head injures associated with isolated facial fractures.

How well did you know this?
1
Not at all
2
3
4
5
Perfectly
32
Q

A 4-year-old girl is brought to the emergency department for evaluation of mandibular fractures 12 hours after being kicked in the face by a horse. Physical examination shows an anterior open bite and bilateral crossbites. CT scans are shown. Which of the following is the most appropriate management at this time?

A) Closed reduction of all mandibular fractures and maxillomandibular fixation
B) Open reduction and internal fixation of all mandibular fractures
C) Open reduction and internal fixation of the parasymphysial and condylar fractures, closed reduction of the ramal fracture, and maxillomandibular fixation
D) Open reduction and internal fixation of the parasymphysial fracture, closed reduction of the condylar and ramal fractures, and maxillomandibular fixation
E) Observation only

A

The correct response is Option D.

In comparison to adult fractures, pediatric fractures are treated more conservatively due to the stage of mixed dentition, the elasticity of the craniofacial skeleton, and the potential for remodeling of the bone and fracture site with growth. Consideration must be given to the effects of invasive surgery on growth of the jaw. Many surgeons believe that conservative management prevents further growth disturbance.

Given the extent of fractures and displacement of the parasymphysial fracture, observation alone is not an option. Closed reduction of all of the fractures followed by maxillomandibular fixation (MMF) will not yield a stable result due to the unfavorable forces displacing the right hemi mandible. Open reduction and internal fixation (ORIF) of all fractures is not possible due to the intracapsular nature of the condylar fracture. The fragments are too small to internally fixate. ORIF of the displaced parasymphysial fracture and closed reduction of the condylar and ramal fractures is the most appropriate management. The occlusion is restored, and bone anchor screws are used to obtain MMF. Due to the child’s dentition, arch bars are not an option. MMF can be accomplished with a custom splint and drop wires with circummandibular wires or with bone anchor screws. Care needs to be taken to place the screws to avoid the dentition. MMF is typically removed at 3 to 4 weeks in this age group to prevent any ankylosis of the temporomandibular joint. If a titanium plate is used to fixate the parasymphysial fracture, removal of this 2 to 3 months following surgery is also recommended to prevent any growth restriction and embedding of the plate.

How well did you know this?
1
Not at all
2
3
4
5
Perfectly
33
Q

A 25-year-old man comes to the office for treatment of malocclusion 6 days after he was involved in an altercation. Physical examination shows right-sided facial swelling. X-ray study shows an unfavorable fracture through the angle of the right mandible. No other associated injuries are noted. Which of the following is the most appropriate treatment?

A) Closed reduction and maxillomandibular fixation
B) Open reduction and maxillomandibular fixation
C) Open reduction and rigid fixation
D) Open reduction, wire fixation, and maxillomandibular fixation
E) Observation only

A

The correct response is Option C.

Mandible fractures are a frequent injury because of the mandible’s prominence and relative lack of support. Numerous investigators have reported studies on populations on all continents; fractures of the mandible have been reported to account for 36 to 70% of all maxillofacial fractures. All reports apparently show a higher frequency in males aged 21 to 30 years. In patients with mandible fractures, 53% of patients had unilateral fractures, 37% of the patients had two fractures, and 9% had three or more fractures. Fractures sustained in altercations – low-energy fractures – tend to have single, simple patterns.

The indications for closed versus open reduction have changed dramatically over the last century. The ability to treat fractures with open reduction and rigid internal fixation (ORIF) has dramatically revolutionized the approach to mandibular fractures.

Traditionally, closed reduction (CR) and ORIF with wire osteosynthesis have required an average of 6 weeks of immobilization by maxillomandibular fixation for satisfactory healing. Difficulties associated with this extended period of immobilization include airway problems, poor nutrition, weight loss, poor hygiene, phonation difficulties, insomnia, social inconvenience, patient discomfort, work loss, and difficulty recovering normal range of jaw function.

In contrast, rigid and semirigid fixation of mandible fractures allow early mobilization and restoration of jaw function, airway control, improved nutritional status, improved speech, better oral hygiene, patient comfort, and an earlier return to the workplace.

How well did you know this?
1
Not at all
2
3
4
5
Perfectly
34
Q

A 55-year-old woman is referred to the office by her dentist because of a 6-week history of exposed intraoral bone. She takes zoledronic acid for osteoporosis. Physical examination shows a 1-cm ulceration of gingiva with exposed necrotic bone adjacent to the right premolar. No infection or fistulization is noted. In addition to meticulous oral hygiene, which of the following is the most appropriate management?

A) Administration of prophylactic oral antibiotics
B) Curettage and bone grafting
C) Dental extraction
D) Segmental resection
E) Observation only

A

The correct response is Option E.

The most appropriate management is observation only. The clinical vignette illustrates a case of Stage I bisphosphonate-related osteonecrosis of the jaw (BRONJ). These patients are typically asymptomatic, with the exception of exposed and/or necrotic bone. Antibiotics are not recommended unless there is infection (Stage II or III). Stage II BRONJ features exposed and/or necrotic bone with pain and local infection.

Curettage and bone grafting is not appropriate.

Dental extractions are risky and may trigger exacerbation. In cases of serious tooth decay, endodontics (root canal) and crown amputation is preferred.

Segmental resection is reserved for Stage III BRONJ. In general, the need for surgery is guided by the severity of the stage. Stage III BRONJ is characterized by exposed and/or necrotic bone with pain, infection, and the presence of another complication, such as osteolysis extending from the superior to the inferior border of the mandible, pathologic fracture, or extraoral fistula.

How well did you know this?
1
Not at all
2
3
4
5
Perfectly
35
Q

A 2-month-old male infant is brought to the office because of mid face hypoplasia, craniosynostosis, and bilateral hand and foot anomalies. A photograph of the left foot is shown. This patient most likely has which of the following syndromes?

A) Apert
B) Crouzon
C) Goldenhar
D) Nager
E) Treacher Collins

A

The correct response is Option A.

The patient described has Apert syndrome. This autosomal dominant syndrome is characterized by bicoronal craniosynostosis that leads to turribrachycephaly, mid face hypoplasia, and complex hand and feet syndactyly. Patients with Crouzon syndrome, an autosomal dominant disorder, typically have craniosynostosis involving the coronal, sagittal, and lambdoid sutures, as well as turribrachycephaly. Other findings include mid face hypoplasia, exorbitism, and proptosis. The extremities are normal.

Goldenhar syndrome, or oculoauriculovertebral dysplasia, involves asymmetry of the hard and soft tissues of the face. This condition is most commonly unilateral but may be seen bilaterally in some patients. Manifestations of this syndrome include hypoplasia involving the mandible and underlying soft tissues of the face, epibulbar dermoids, and varied degrees of microtia on the affected side. Most patients have associated vertebral abnormalities. Nager syndrome, or acrofacial dysostosis, is an autosomal recessive disorder characterized by craniofacial and upper extremity abnormalities. Patients with Nager syndrome have hypoplasia of the orbits, zygoma, maxilla, mandible, and soft palate. Auricular defects may also be present. Hypoplasia or agenesis occurs in the radius, thumbs, and metacarpals. Some patients may have radioulnar synostosis and elbow joint deformities. Patients with Treacher Collins syndrome, or mandibular dysostosis, have hypoplasia of the zygoma, maxilla, and mandible, downward slanting of the palpebral fissures, colobomas of the lower eyelids, absence of eyelashes, and auricular defects.

36
Q

A 33-year-old woman is brought to the emergency department after sustaining injuries in a motor vehicle collision. The patient notes pain on opening her mouth. Physical examination shows bilateral facial swelling and loss of posterior facial height. An anterior open bite is also noted. Which of the following additional findings on physical examination is most likely to suggest a bilateral subcondylar fracture in this patient?

A) Bilateral facial numbness
B) Bilateral mastoid ecchymosis
C) Blood in the external auditory canal
D) Clear fluid in the external auditory canal
E) Preauricular pain

A

The correct response is Option E.

Bilateral subcondylar fractures result in premature occlusion of the posterior teeth along with an anterior open bite, loss of posterior facial height, and bilateral facial swelling with pain on mouth opening. These findings occur because the subcondylar fracture interrupts the integrity of the vertical buttress. The lateral pterygoid muscles displace the condylar necks medially and anteriorly, allowing the unopposed vertical action of the temporalis and masseter muscles to shorten the posterior facial height. Swelling bilaterally would be expected in this fracture, and opening would displace the fracture line, causing pain.

Bilateral facial numbness suggests a fracture of the ramus or body as the inferior alveolar nerve traverses these areas. Blood in the external auditory canal suggests a fracture more proximal than subcondylar. Bilateral mastoid ecchymosis and/or clear fluid in the external auditory canal suggest a skull base fracture.

37
Q

A 25-year-old man is brought to the emergency department 2 hours after being punched in the left side of the face. Physical examination shows swelling on the left side of the face. The panoramic x-ray study (Panorex) shown was obtained. Which of the following is the most appropriate management?

A) Extraction of all components of the third left mandibular molar and maxillomandibular fixation (MMF) for 2 weeks
B) Extraction of all components of the third left mandibular molar and MMF for 6 weeks
C) Extraction of all components of the third left mandibular molar, MMF, and open reduction and internal fixation (ORIF)
D) Ligating the third left mandibular molar to the adjacent tooth for stability, MMF, and ORIF
E) Preservation of the third molar, MMF, and ORIF

A

The correct response is Option C.

The x-ray study shows a fracture of the root, thus the tooth and root must be removed. Indications for extraction of a tooth in the line of a fracture include the need for MMF to regain the patient’s occlusion. The left parasymphyseal and comminuted left angle fractures can be managed in a number of ways, but the parasymphyseal fracture requires open reduction and internal fixation to prevent lateral displacement of the left mandibular body by the masseter muscle.

38
Q

A 66-year-old man comes for a follow-up examination 7 months after resection of a T4 N1 M0 squamous cell carcinoma in the region of the retromolar trigone, including alveolectomy, followed by soft-tissue reconstruction with a platysma flap. Postoperatively, he received radiation therapy to the primary tumor site (6 Gy) and to the neck bilaterally (64 Gy). He completed radiation therapy 5 months ago. Examination today shows a malodorous, tender area of exposed, soft bone at the operative site. A panoramic x-ray study (Panorex) is shown. Multiple biopsies are negative for recurrent carcinoma. Which of the following is the most appropriate management?

A ) Long-term intravenous antibiotic therapy

B ) Open reduction and internal fixation

C ) Segmental resection and vascularized tissue transfer

D ) Sequestrectomy

A

The correct response is Option C.

The patient described has osteoradionecrosis of the mandible, a complication that occurs in up to 40% of patients receiving adjuvant radiation therapy for head and neck malignancies caused by hypoxia, hypovascularity, hypocellularity, and impaired collagen synthesis. The traditional definition is an area of exposed, irradiated bone that is nonhealing over 3 months. Treatment depends on the severity of the disease. Debridement and antibiotic therapy, plus or minus

hyperbaric oxygen therapy, with soft-tissue reconstruction as needed, may be curative in up to 90% of cases of osteoradionecrosis limited to the alveolar ridge or mandible superior to the alveolar canal. When more extensive destruction of the mandible is present, or when there is a pathologic fracture, as seen in the scenario described, resection of all the necrotic bone and soft tissue is indicated, followed by reconstruction with vascularized bone and soft tissue. Successful healing occurs in up to 80 to 90% of patients with more extensive disease when treated in this way. Local flaps are of limited use for soft-tissue coverage because of the radiation.

39
Q

A 40-year-old woman is referred for treatment after being found to have an idiopathic fracture on panoramic radiograph (Panorex), performed because of severe pain following a dental procedure. She has been treated for osteopenia related to multiple myeloma. A CT is shown. On evaluation, the patient has chronic pain and exposed intraoral bone. Which of the following is the most likely diagnosis?

A ) Bisphosphonate-related osteonecrosis

B ) Breast cancer metastasis

C ) Infectious osteomyelitis

D ) Myeloma-induced changes to the mandible

E ) Old unhealed fracture from trauma

A

The correct response is Option A.

Bisphosphonate-related osteonecrosis (BRON) is an increasingly reported diagnosis for patients being treated with bisphosphonates for bone loss from Paget disease, osteoporosis, and bone-related cancer. Bisphosphonates are clinically important for the treatment of hypercalcemia of malignancy and can reduce cancer-induced bone pain. The two bisphosphonates approved by the FDA for use in patients with cancer involving bone are pamidronate and zoledronic acid. Because of the high frequency of skeletal involvement in advanced cancers, bisphosphonates are routinely prescribed in the practice of medical oncology, and the incidence in these patients for BRON ranges between 1% and 10%. Because of the common use of these drugs in multiple myeloma, there have been many case reports of patients with BRON. Other diseases with a similar presentation include intraoral conditions including periodontal disease, gingivitis, or mucositis; temporomandibular joint disease; sinusitis; periapical pathology caused by a carious infection; osteoradionecrosis; neuralgia-inducing cavitational osteonecrosis; odontogenic infections leading to osteomyelitis; herpes zoster infection €“associated osteonecrosis; benign sequestration of the lingual plate; or HIV-associated necrotizing ulcerative periodontitis.

For established pathology, there is little need for diagnostic imaging techniques because the presence of exposed bone and associated symptoms such as pain, swelling, paresthesia, suppuration, and soft-tissue ulceration can be detected without them. However, early identification of BRON, although more challenging, is potentially important for patient care and prevention of disease. A Panorex or CT combined with clinical history, examination, and biopsy are adequate.

Recommendations for patients with established BRON include oral antimicrobial rinses, such as 0.12% chlorhexidine digluconate. Systemic antibiotic therapy may be prescribed if there is evidence of infection. Establishing and maintaining an €œinfection-free € oral environment is especially important for patients with multiple myeloma who are being considered for stem cell transplantation. Surgical treatment should be conservative or delayed. Removal of sharp bone edges is recommended to prevent trauma to adjacent soft tissues. Loose segments of bony sequestra should be removed without exposing uninvolved bone. Segmental jaw resection may be required for symptomatic patients with large segments of necrotic bone or pathologic fracture, and with this, the potential need for vascularized tissue. No published data have established that stopping bisphosphonates will promote resolution of BRON. Although some have advocated hyperbaric oxygen therapy, the efficacy of this approach has not been established.

There have been many case reports of breast cancer metastatic to the mandible; it must be considered in any woman with severe changes to this bone. Multiple myeloma is most frequently seen in vertebrae and secondarily in long bones. Its presence in the jaw is extremely rare, and when it is seen, it is usually in the mandibular angle and ramus.

40
Q

A 32-year-old man comes to the emergency department because he has had jaw pain and difficulty closing his mouth after he was punched in the face 30 minutes ago. A panoramic radiograph (Panorex) of the lower face is shown. Which of the following clinical findings is consistent with this fracture pattern?

A ) Anterior crossbite

B ) Fracture of tooth No. 31

C ) Maxillary occlusal cant

D ) Paresthesia of right upper lip

E ) Posterior open bite on the left

A

The correct response is Option E.

A right mandibular angle fracture is shown. Foreshortening of the height on the right from the fracture and powerful influence of the masseter will likely result in posterior open bite on the contralateral side.

Anterior crossbite occurs in maxillary hypoplasia and prognathism conditions and is not a result of the fracture in the patient described.

Tooth No. 32 is fractured in the Panorex shown. Standard numbering of the dentition of the maxillary dental arch from right to left is No. 1 to No. 16. The mandibular dental arch from left to right is No. 17 through No. 32.

Maxillary occlusal cant occurs congenitally or with maxillary fracture.

Paresthesias of the upper lip are not associated with mandibular fracture through the inferior alveolar nerve. This fracture would be consistent with paresthesia of the right lower lip.

41
Q

A 56-year-old woman with type 1 diabetes mellitus has a six-month history of a slowly enlarging, painless mass in the mandible. She is edentulous. Physical examination shows a 4-cm mass on both the buccal and lingual portions of the alveolus with intact mucosa. A panoramic radiograph (Panorex) is shown. Biopsy confirms ameloblastoma. Which of the following is the most definitive management?

A ) Conservative segmental resection of the mandible, followed by nonvascularized iliac crest cortical bone grafting

B ) Curettage and cancellous bone grafting

C ) Segmental resection of the mandible with wide margins and cervical lymph node dissection, followed by vascularized free fibular bone grafting

D ) Segmental resection of the mandible with wide margins and cervical lymph node

dissection, followed by vascularized free fibular bone grafting and adjuvant radiation

therapy

E ) Segmental resection of the mandible with wide margins, followed by vascularized free fibular bone grafting

A

The correct response is Option E.

The history and radiographic findings in the patient described are most consistent with multicystic ameloblastoma. This odontogenic tumor commonly presents as a painless enlarging mass in the mandible or, less commonly, in the maxilla. The typical radiographic finding is a multilocular lucency with preservation of the cortex, though the cortex may be thinned. There are subcategories including peripheral (extraosseous) and central (intraosseous), which may be unicystic or multicystic, as in the radiograph shown. Extraosseous lesions may have no radiographic findings. This tumor goes beyond the boundaries seen on radiograph.

Wide resection is necessary for multicystic lesions to prevent recurrence. The sizeable defect resulting from wide resection in the patient described would be best treated with vascularized bone graft, especially in light of her diabetes.

Curettage or conservative resection of this tumor would result in recurrence in 50% of patients. Conservative resection may have higher cure rates for unicystic lesions. The lack of mucosal ulceration or cortical bone erosion in a lesion this size point away from a malignancy; therefore, cervical lymph node dissection and adjuvant radiation therapy would not be indicated.

42
Q

A 5-year-old girl is brought to the emergency department immediately after falling from a swing set and hitting her chin on the cement pavement. Physical examination shows a laceration of the chin and an anterior open bite. Radiographs show bilateral subcondylar fractures with medial displacement of the right condylar neck and comminution of the left condylar head. A good dental relation is established while using anesthesia. Which of the following is the most appropriate management?

A ) Intermaxillary fixation for one to two weeks

B ) Intermaxillary fixation for four to six weeks

C ) Open reduction and internal fixation of the right and left fractures followed by early range of motion exercises

D ) Open reduction and internal fixation of the right subcondylar fracture followed by intermaxillary fixation for one to two weeks

E ) Open reduction and internal fixation of the right subcondylar fracture followed by intermaxillary fixation for four to six weeks

A

The correct response is Option A.

Open reduction of a fracture of the mandibular condyle is not commonly performed because the procedure may be complicated, and closed reduction is usually sufficient. However, open reduction of the condyle is indicated in the following four situations:

Displacement into the middle cranial fossa

Impossibility of obtaining adequate dental occlusion by closed reduction

Lateral extracapsular displacement of the condyle

Invasion by a foreign body (eg, a bullet from a gunshot wound)

Because the condylar head is not displaced in the patient described, open reduction and internal fixation of the right subcondylar fracture is not indicated. Intermaxillary fixation should be applied and remain in place for one to two weeks after the surgery. Subsequently, active and passive physical therapy of the mandible should be performed to work the mandible and remold the subcondylar union. This is especially important because of the comminuted intracapsular fracture on the contralateral side. Intermaxillary fixation applied for an extended period (greater than four weeks) may result in postoperative ankylosis. This is very difficult to treat. Releasing the intermaxillary fixation at two weeks may cause a delayed crossbite or anterior open bite. Both may be treated secondarily with a sagittal split osteotomy.

43
Q

A patient with bilateral displaced subcondylar fractures is most likely to have which of the following occlusal patterns?

A ) Anterior crossbite

B ) Anterior open bite

C ) Posterior crossbite

D ) Posterior open bite

A

The correct response is Option B.

When a patient sustains a bilateral subcondylar fracture, there is the possibility of shortening of the posterior mandibular height. This shortening is secondary to telescoping of the condylar fragments and muscular pull of the boney components.

A premature occlusal contact of the posterior occlusion occurs, resulting in an anterior open bite.

44
Q

A 6 year old boy is evaluated because of severe pain on opening his mouth. One week ago, he hit his chin in a fall. Physical examination shows chin deviation to the right and premature contact of the molar region on the right. Facial radiographs are ordered. The most likely cause of these findings is fracture of which of the following segments of the mandible?

(A) Angle

(B) Body

(C) Condyle

(D) Ramus

(E) Symphysis

A

The correct response is Option C.

Mandibular condyle fractures are the most common facial fractures seen in children. In a recent review of more than 1250 pediatric maxillofacial fractures, condylar injuries constituted 34.9% of all mandible fractures; however, the angle (17.7%), body (14.9%), ramus (8%), and symphysis (9%) are also important sites of potential mandible fracture.

45
Q

Which of the following regions of the adult mandible has the highest incidence of fracture?

(A) Angle

(B) Body

(C) Coronoid

(D) Ramus

(E) Symphysis

A

The correct response is Option A.

The angle of the mandible has the highest incidence of fracture €”up to 30%. Incidence of fracture is 15% to 20% for the body and parasymphysis. The presence of an isolated symphyseal or parasymphyseal fracture should alert the surgeon to the possibility of a second fracture near the angle.

46
Q

Which of the following muscles controls depression and protrusion of the mandible?

(A) Buccinator

(B) Digastric

(C) Lateral pterygoid

(D) Masseter

(E) Temporalis

A

The correct response is Option C.

The muscles of mastication are involved in the process of biting and chewing. These are all inserted upon the mandible and innervated by the mandibular division of the trigeminal (V) nerve. The lateral pterygoid muscle originates from the greater wing of the sphenoid, the inferotemporal crest, and the lateral pterygoid plate and inserts on the neck of the mandibular condyle and articular disc of the temporomandibular joint. Its action is to depress, protrude, and move the mandible from side to side. The masseter muscle originates from the zygomatic arch and inserts on the mandibular angle, ramus, and condyle. Its action is to close the jaw. The medial pterygoid muscle originates on the medial aspect of the lateral pterygoid plate of the sphenoid in the tuberosity of the maxilla. It inserts on the medial surface of the ramus of the mandible and acts to close the jaw. The temporalis muscle originates from the temporal fascia and entire temporal fossa. It inserts on the coronoid process and the anterior border of the ramus of the mandible and acts to close and retract the jaw. The buccinator muscle is innervated by the facial nerve (VII). Although this muscle aids in mastication by compressing the cheek, thus holding food under the teeth, it is not considered a muscle of mastication.

47
Q

A 20-year-old man comes to the office for consultation regarding malocclusion. On physical examination, the mandibular incisors are anterior to the maxillary incisors. The mesial buccal cusp of the maxillary first molar lies distal to the buccal groove of the mandibular first molar. These findings are characteristic of which of the following Angle classifications?

(A) Angle class I

(B) Angle class II, division I

(C) Angle class II, division II

(D) Angle class III

A

The correct response is Option D.

Angle class I, or normal occlusion, is characterized as having the mesial buccal cusp of the upper first molar occluding in the buccal groove of the mandibular first molar. In Angle class II, both divisions I and II, the mandibular dentition is distal to its class I position. Class II, division 1 is lingually inclined, and Class II, division 2 is labially inclined. In Angle class III malocclusion, the mandibular molar is anterior to its normal position with the maxillary molar.

48
Q

A 46-year-old man comes to the office because he has pain in the jaw and trismus after being involved in a motor vehicle collision two days ago. Radiographs show a mandibular fracture. In adults, the normal range of vertical mandibular opening is closest to which of the following?

(A) 11 to 20 mm

(B) 21 to 30 mm

(C) 31 to 40 mm

(D) 41 to 50 mm

(E) 51 to 60 mm

A

The correct response is Option D.

In adults, the vertical mandibular opening measured from maxillary incisal edge to mandibular incisal edge (interincisal distance) typically ranges from 40 to 50 mm. In addition, normal range of motion of the mandible includes lateral jaw excursion (measured at the midline incisor) to 10 mm on each side.

Decreased mandibular opening may indicate dysfunction of the temporomandibular joint (TMJ) or surrounding soft tissues. Patients who may potentially have internal derangement of the TMJ also may experience painless clicking when opening the mouth.

49
Q

In planning open reduction and internal fixation in a patient with a low subcondylar neck fracture, which of the following extraoral incisions provides the safest and most versatile exposure to the fracture site?
(A) Postauricular
(B) Preauricular
(C) Retromandibular
(D) Submandibular

A

The correct response is Option C.

The retromandibular incision provides the safest and most versatile exposure for open reduction and internal fixation of submandibular fractures. When compared with the subcondylar and preauricular incisions, there is significantly less injury to the marginal mandibular, temporal, and zygomatic branches of the facial nerve.

The retromandibular incision allows access superiorly to the coronoid notch and inferiorly to the angle of the mandible. The addition of a transfacial trocar to this approach facilitates access to higher level subcondylar fractures as well. The preauricular, postauricular, and submandibular incisions provide a more limited view of low subcondylar fractures.

50
Q

A 5-year-old child has malocclusion and limited opening of the mouth after falling from playground equipment. A CT scan is shown (condylar fracture). Which of the following is the most likely long-term sequela of this patient’s injury?

(A) Bimaxillary prognathism
(B) Mandibular hypoplasia
(C) Mandibular prognathism
(D) Maxillary hypoplasia
(E) Maxillary prognathism

A

The correct response is Option B.

This child is at increased risk for mandibular hypoplasia. The condyle serves as a growth center for the mandible and contributes primarily to vertical growth. The condylar cartilage is a site of secondary passive growth dependent on forces acting on it, notably the medial and lateral pterygoid muscles. Pediatric condylar fractures generally remodel and do not often cause growth disturbance. However, the thin, localized functional matrix of the condyle may disallow normal mandibular growth after it has been injured and may result in unilateral or bilateral hypoplasia depending on the injury. Pediatric mandibular fractures are frequently treated conservatively via closed reduction and short periods of maxillomandibular fixation.

51
Q

A 12-year-old boy is brought to the emergency department after he fell while riding his bike and landed on the chin. Panorex radiographs show a minimally displaced fracture of the high right condylar neck and an open left parasymphyseal fracture. Which of the following is the most effective management?
(A) Observation with serial radiographs and restriction to soft diet
(B) Intermaxillary fixation with infraorbital and circummandibular wires for four weeks
(C) Intermaxillary fixation with arch bars for four weeks
(D) Open reduction with internal fixation of the parasymphyseal fracture and arch bars for two weeks
(E) Open reduction with internal fixation of both the parasymphyseal fracture and the fracture of the high condylar neck

A

The correct response is Option D.

Observation alone is inadequate management of the parasymphyseal fracture. This child presents in the late phase of the mixed dentition and should have enough adult dentition in place to secure the arch bars. Wires to reduce the fractures will not provide as much stability as arch bars. This technique, however, is useful in the child who does not yet have enough adult dentition to secure the arch bars.

Immobilization for a short period (i.e., two weeks) is the appropriate management of the condylar neck fracture. This will help to allow the fractures to become stable enough to maintain the reduction once movement is instituted. Early movement helps to decrease the risk of ankylosis of the temporal mandibular joint. Extended immobilization to allow for healing of the parasymphyseal fracture will increase the risk of ankylosis of the temporomandibular joint. Open reduction and internal fixation (ORIF) of the parasymphyseal fracture will ensure an anatomic reduction of a stable skeletal unit. This, in turn, will decrease the risk of infection and nonunion.

ORIF of the condylar neck fracture is not indicated because the fracture is minimally displaced. Conservative management of these fractures is well accepted and has stood the test of time. Open reduction would incur the risks of injury of the facial nerve.

52
Q

A 22-year-old man sustains a left subcondylar fracture of the mandible during a motor vehicle collision. On CT scan, the condyle is displaced medially and anteriorly. This displacement is most likely caused by tension from which of the following muscles?
(A) Medial pterygoid
(B) Lateral pterygoid
(C) Masseter
(D) Mylohyoid
(E) Temporalis

A

The correct response is Option B.

The inferior belly of the lateral pterygoid originates from the lateral pterygoid plate and inserts onto the scaphoid fossa of the condyle and joint capsule. The superior belly of the lateral pterygoid muscle originates from the sphenoid and inserts on the temporomandibular joint. The effect of the lateral pterygoid muscle is to displace the condyle medially and anteriorly in fractures of the condylar neck. It also tends to displace the meniscus anteriorly. The muscles inserting directly on the mandible exert significant forces on fracture fragments. An understanding of their direction of pull and insertions is important in the proper reduction and fixation of mandibular fractures. All the muscles of mastication serve to elevate and protrude the mandible. The elevators include the masseter, medial pterygoid, and temporalis. The temporalis inserts onto the coronoid process and the superior aspect of the external oblique line. The masseter inserts onto the lateral aspect of the mandibular angle while the medial pterygoid inserts on the medial aspect of the mandibular angle. None of these muscles directly affect the condyle. The mylohyoid muscle inserts on the body of the mandible, displacing segmental body fractures medially.

53
Q

A 22-year-old man sustains a transverse, noncomminuted fracture of the right mandibular angle when he is struck in the face during a fistfight. Which of the following interventions best adheres to Champy’s principle for management of this fracture?
(A) Dynamic compression plate with bicortical screws on the inferior edge of the mandible and a superior tension band
(B) Dynamic compression plate with bicortical screws and a mandibular arch bar
(C) Lag screw
(D) Miniplate with monocortical screws along the external oblique ridge
(E) Reconstruction plate with bicortical screws

A

The correct response is Option D.

Champy’s principles for fracture management call for placement of miniplates along the lines of tension in the mandible at the site of the fracture. Because compression is not necessary, the miniplates can be anchored with monocortical screws. Based on the muscular forces pulling on the mandible, Champy determined that, anterior to the canine tooth, two miniplates are needed to control the rotational forces of the genial and digastric muscles; posterior to the canine tooth, just one miniplate is required.

In a 10-year review examining various methods for treating fractures of the mandibular angle, Ellis concluded that use of a single 2.0-mm noncompression miniplate was associated with fewer complications than a double-plate system (one using two compression or noncompression plates) or a reconstruction plate.

54
Q

A 26-year-old man comes to the office because he has pain in the mandible seven days after undergoing open reduction and internal fixation of a fracture of the mandible. Physical examination shows infection in the submandibular space. Which of the following teeth are the most likely source of this infection?
(A) Mandibular canines
(B) Mandibular central incisors
(C) Mandibular first and second premolars
(D) Mandibular second and third molars
(E) Maxillary second and third molars

A

The correct response is Option D.

The submandibular space is located inferolateral to the mylohyoid muscle and superior to the hyoid bone. The contents of the submandibular space include the submandibular gland, lymph nodes, the facial vein and artery, and the inferior loop of the hypoglossal (XII) nerve. Anteriorly, the submandibular space communicates with the submental space and posteriorly with the pharyngeal space. The sublingual space is located superomedial to the mylohyoid muscle. Involvement of the submandibular space is produced principally by infections of the second and third mandibular molars because of the more superior position of the mylohyoid ridge on the mandible posteriorly, which places the root apices of the second and third molars beneath the mylohyoid muscle. Infections of the maxillary molars, when they extend through the buccal cortical plates above the attachments of the buccinator muscle, can present as infections of the buccal space. Infections from the anterior mandibular teeth (anterior to the second molar) usually drain above the mylohyoid muscle into the sublingual space.

55
Q

The application of a locking reconstruction plate to a comminuted mandibular fracture is LEAST likely to cause which of the following?

(A) Decreased bone resorption
(B) More difficulty in contouring the plate
(C) Hardware failure
(D) Hardware-related infection
(E) Malocclusion

A

The correct response is Option E.

Use of a locking reconstruction bone plate has been shown to decrease postoperative malocclusion after a comminuted fracture of the mandible. A conventional (nonlocking) bone plate requires precise adaptation of the plate to the underlying bone. Without intimate contact, the bone is drawn toward the plate when the screws are tightened, altering the position of the osseous segments and the occlusal relationship. However, a locking bone plate does not require intimate contact of plate to bone because the bony segments are secured by screws that are locked to the plate. This makes it less likely for screw insertion to alter the reduction and, ultimately, the occlusion.

Cortical compression, blood supply disruption, and associated bone resorption occur less frequently with locking plates than with standard reconstruction plates. Difficulty in plate contouring is less likely to occur with locking plates because they require less precise bending than do conventional plates, which depend on intimate bony contact for stability. No increase in hardware failure has been noted with locking plates. In fact, screws in locking plates are less likely to become loose than those in standard reconstruction plates. The rate of hardware-related infection with locking plates is similar to the rate with standard reconstruction plates.

56
Q

In planning open reduction and internal fixation in a patient with a low subcondylar neck fracture, which of the following extraoral incisions provides the safest and most versatile exposure to the fracture site?
(A) Postauricular
(B) Preauricular
(C) Retromandibular
(D) Submandibular

A

The correct response is Option C.

The retromandibular incision provides the safest and most versatile exposure for open reduction and internal fixation of submandibular fractures. When compared with the subcondylar and preauricular incisions, there is significantly less injury to the marginal mandibular, temporal, and zygomatic branches of the facial nerve.

The retromandibular incision allows access superiorly to the coronoid notch and inferiorly to the angle of the mandible. The addition of a transfacial trocar to this approach facilitates access to higher level subcondylar fractures as well. The preauricular, postauricular, and submandibular incisions provide a more limited view of low subcondylar fractures.

57
Q

A 5-year-old child has malocclusion and limited opening of the mouth after falling from playground equipment. A CT scan is shown. Which of the following is the most likely long-term sequela of this patient’s injury?

(A) Bimaxillary prognathism
(B) Mandibular hypoplasia
(C) Mandibular prognathism
(D) Maxillary hypoplasia
(E) Maxillary prognathism

A

The correct response is Option B.

This child is at increased risk for mandibular hypoplasia. The condyle serves as a growth center for the mandible and contributes primarily to vertical growth. The condylar cartilage is a site of secondary passive growth dependent on forces acting on it, notably the medial and lateral pterygoid muscles. Pediatric condylar fractures generally remodel and do not often cause growth disturbance. However, the thin, localized functional matrix of the condyle may disallow normal mandibular growth after it has been injured and may result in unilateral or bilateral hypoplasia depending on the injury. Pediatric mandibular fractures are frequently treated conservatively via closed reduction and short periods of maxillomandibular fixation.

58
Q

A 12-year-old boy is brought to the emergency department after he fell while riding his bike and landed on the chin. Panorex radiographs show a minimally displaced fracture of the high right condylar neck and an open left parasymphyseal fracture. Which of the following is the most effective management?

(A) Observation with serial radiographs and restriction to soft diet
(B) Intermaxillary fixation with infraorbital and circummandibular wires for four weeks
(C) Intermaxillary fixation with arch bars for four weeks
(D) Open reduction with internal fixation of the parasymphyseal fracture and arch bars for two weeks
(E) Open reduction with internal fixation of both the parasymphyseal fracture and the fracture of the high condylar neck

A

The correct response is Option D.

Observation alone is inadequate management of the parasymphyseal fracture. This child presents in the late phase of the mixed dentition and should have enough adult dentition in place to secure the arch bars. Wires to reduce the fractures will not provide as much stability as arch bars. This technique, however, is useful in the child who does not yet have enough adult dentition to secure the arch bars.

Immobilization for a short period (i.e., two weeks) is the appropriate management of the condylar neck fracture. This will help to allow the fractures to become stable enough to maintain the reduction once movement is instituted. Early movement helps to decrease the risk of ankylosis of the temporal mandibular joint. Extended immobilization to allow for healing of the parasymphyseal fracture will increase the risk of ankylosis of the temporomandibular joint. Open reduction and internal fixation (ORIF) of the parasymphyseal fracture will ensure an anatomic reduction of a stable skeletal unit. This, in turn, will decrease the risk of infection and nonunion.

ORIF of the condylar neck fracture is not indicated because the fracture is minimally displaced. Conservative management of these fractures is well accepted and has stood the test of time. Open reduction would incur the risks of injury of the facial nerve.

59
Q

A 22-year-old man sustains a left subcondylar fracture of the mandible during a motor vehicle collision. On CT scan, the condyle is displaced medially and anteriorly. This displacement is most likely caused by tension from which of the following muscles?
(A) Medial pterygoid
(B) Lateral pterygoid
(C) Masseter
(D) Mylohyoid
(E) Temporalis

A

The correct response is Option B.

The inferior belly of the lateral pterygoid originates from the lateral pterygoid plate and inserts onto the scaphoid fossa of the condyle and joint capsule. The superior belly of the lateral pterygoid muscle originates from the sphenoid and inserts on the temporomandibular joint. The effect of the lateral pterygoid muscle is to displace the condyle medially and anteriorly in fractures of the condylar neck. It also tends to displace the meniscus anteriorly. The muscles inserting directly on the mandible exert significant forces on fracture fragments. An understanding of their direction of pull and insertions is important in the proper reduction and fixation of mandibular fractures. All the muscles of mastication serve to elevate and protrude the mandible. The elevators include the masseter, medial pterygoid, and temporalis. The temporalis inserts onto the coronoid process and the superior aspect of the external oblique line. The masseter inserts onto the lateral aspect of the mandibular angle while the medial pterygoid inserts on the medial aspect of the mandibular angle. None of these muscles directly affect the condyle. The mylohyoid muscle inserts on the body of the mandible, displacing segmental body fractures medially.

60
Q

A 22-year-old man sustains a transverse, noncomminuted fracture of the right mandibular angle when he is struck in the face during a fistfight. Which of the following interventions best adheres to Champy’s principle for management of this fracture?
(A) Dynamic compression plate with bicortical screws on the inferior edge of the mandible and a superior tension band
(B) Dynamic compression plate with bicortical screws and a mandibular arch bar
(C) Lag screw
(D) Miniplate with monocortical screws along the external oblique ridge
(E) Reconstruction plate with bicortical screws

A

The correct response is Option D.

Champy’s principles for fracture management call for placement of miniplates along the lines of tension in the mandible at the site of the fracture. Because compression is not necessary, the miniplates can be anchored with monocortical screws. Based on the muscular forces pulling on the mandible, Champy determined that, anterior to the canine tooth, two miniplates are needed to control the rotational forces of the genial and digastric muscles; posterior to the canine tooth, just one miniplate is required.

In a 10-year review examining various methods for treating fractures of the mandibular angle, Ellis concluded that use of a single 2.0-mm noncompression miniplate was associated with fewer complications than a double-plate system (one using two compression or noncompression plates) or a reconstruction plate.

61
Q

A 26-year-old man comes to the office because he has pain in the mandible seven days after undergoing open reduction and internal fixation of a fracture of the mandible. Physical examination shows infection in the submandibular space. Which of the following teeth are the most likely source of this infection?

(A) Mandibular canines
(B) Mandibular central incisors
(C) Mandibular first and second premolars
(D) Mandibular second and third molars
(E) Maxillary second and third molars

A

The correct response is Option D.

The submandibular space is located inferolateral to the mylohyoid muscle and superior to the hyoid bone. The contents of the submandibular space include the submandibular gland, lymph nodes, the facial vein and artery, and the inferior loop of the hypoglossal (XII) nerve. Anteriorly, the submandibular space communicates with the submental space and posteriorly with the pharyngeal space. The sublingual space is located superomedial to the mylohyoid muscle. Involvement of the submandibular space is produced principally by infections of the second and third mandibular molars because of the more superior position of the mylohyoid ridge on the mandible posteriorly, which places the root apices of the second and third molars beneath the mylohyoid muscle. Infections of the maxillary molars, when they extend through the buccal cortical plates above the attachments of the buccinator muscle, can present as infections of the buccal space. Infections from the anterior mandibular teeth (anterior to the second molar) usually drain above the mylohyoid muscle into the sublingual space.

62
Q

The application of a locking reconstruction plate to a comminuted mandibular fracture is LEAST likely to cause which of the following?

(A) Decreased bone resorption
(B) More difficulty in contouring the plate
(C) Hardware failure
(D) Hardware-related infection
(E) Malocclusion

A

The correct response is Option E.

Use of a locking reconstruction bone plate has been shown to decrease postoperative malocclusion after a comminuted fracture of the mandible. A conventional (nonlocking) bone plate requires precise adaptation of the plate to the underlying bone. Without intimate contact, the bone is drawn toward the plate when the screws are tightened, altering the position of the osseous segments and the occlusal relationship. However, a locking bone plate does not require intimate contact of plate to bone because the bony segments are secured by screws that are locked to the plate. This makes it less likely for screw insertion to alter the reduction and, ultimately, the occlusion.

Cortical compression, blood supply disruption, and associated bone resorption occur less frequently with locking plates than with standard reconstruction plates. Difficulty in plate contouring is less likely to occur with locking plates because they require less precise bending than do conventional plates, which depend on intimate bony contact for stability. No increase in hardware failure has been noted with locking plates. In fact, screws in locking plates are less likely to become loose than those in standard reconstruction plates. The rate of hardware-related infection with locking plates is similar to the rate with standard reconstruction plates.

63
Q

A 67-year-old man has an ulcerated lesion of the anterior floor of the mouth with exposed, desiccated mandible one year after undergoing surgical resection and radiotherapy for squamous cell carcinoma of the anterior floor of the mouth. Initial recovery from the procedure was uncomplicated. Pathologic evaluation of a specimen obtained on excisional biopsy shows osteoradionecrosis. Radiation doses greater than 6500 cGy and which of the following are the most likely precipitating factors in this patient?

(A) Dental caries
(B) Dental implants
(C) Edentulous mandible
(D) Oral candidiasis
(E) Xerostomia

A

The correct response is Option A.

Osteoradionecrosis (ORN) is relatively uncommon. However, the risk of ORN increases when the radiation dosage to the mandible exceeds 6500 cGy. Although up to 30% of cases of ORN reportedly arise spontaneously, most reports note dental caries and extraction sites as precipitating factors. Acute and chronic periodontal disease in mandibulotomy sites can also lead to ORN.

Dental implants, an edentulous mandible, oral candidiasis, and xerostomia may all be seen in cases of mandibular reconstruction and radiation. However, they do not increase the risk of developing ORN.

Traditional treatment of ORN includes surgical debridement and antibiotic therapy if infection is present. In advanced, extensive ORN, hyperbaric oxygen (HBO) therapy may be used as an adjunct. However, controversy exists about using HBO therapy instead of surgical resection and reconstruction of the mandible. The decision to use HBO therapy should be made on an individual basis and with the understanding that necrotic bone with sequestrum is unlikely to heal with HBO therapy.

64
Q

Which of the following terms best describes the type of occlusion in which the upper central incisor lies anterior to the lower central incisor in the sagittal plane?

(A) Buccal crossbite
(B) Lingual crossbite
(C) Open bite
(D) Overbite
(E) Overjet

A

The correct response is Option E.

Overjet is a horizontal measurement that refers to the distance between the incisal aspect of the maxillary incisors and the incisal aspect of the mandibular incisors with the teeth in centric occlusion. When the upper central incisor lies anterior to the lower central incisor in the sagittal plane, this is known as overjet.
In contrast, overbite is a vertical measurement referring to the distance between the maxillary incisor edge and the mandibular incisor edge with the teeth in centric occlusion. An overbite or deep bite is one in which the upper central incisor overrides the lower central significantly in the vertical dimension.

Buccal and lingual crossbite refer to the positioning of the mandibular molars with respect to the maxillary molars in the transverse plane.

Open bite occurs when the maxillary and mandibular teeth fail to contact. This can occur at any point in the dentition.

65
Q

Removal of a tooth in a fracture line of the mandible is indicated in a patient with which of the following conditions?

(A) Cavities in the tooth
(B) Fracture of the root of the tooth
(C) Loose tooth
(D) Multiple fractures of the mandible
(E) Periodontal disease

A

The correct response is Option B.

Indications for removal of teeth in mandibular fractures include fracture of the root of the tooth, severe loosening of the tooth in presence of chronic periodontal disease, extensive periodontal injury and broken alveolar walls, and displacement of teeth from their alveolar socket. Periodontal disease alone is not an indication for tooth removal. Multiple fractures of the mandible are also not an indication for tooth removal because the teeth usually are needed for intermaxillary fixation prior to open reduction and internal fixation of the fractures. History of caries would warrant a referral to a dentist to ascertain whether any intervention would be required but would not necessitate removal of that tooth at the time of fracture management. Loose tooth is seen in most cases of mandibular fracture but is addressed by proper alignment and reduction of all fractures.

66
Q

In distraction osteogenesis of the mandible using an external distractor, successful formation of the bone is most dependent on which of the following?

(A) Consolidation period of two weeks
(B) Distraction rate of 1 mm per day
(C) Lag period of three days before initiation of distraction
(D) Stable fixation of the bone
(E) Supraperiosteal dissection of the bone

A

The correct response is Option D.

In distraction osteogenesis of the mandible using an external distractor, the most important element in successful formation of the bone is adequate stabilization of the bone edges. Unstable fixation of the bone allows excessive motion, which can result in a fibrous union.

A consolidation period of two weeks is inadequate. A period of at least four to six weeks usually is needed before the distraction devices can be safely removed. Although a rate of 1 mm per day commonly is used in mandibular distraction, rates of 2 mm or more per day have been shown to be successful in mandibular distraction, particularly when an external device is used. Most surgeons use a lag period before beginning distraction. However, a lag period has never been demonstrated to be necessary in craniofacial distraction. Supraperiosteal dissection of the bone is important in distraction of the extremities but has not been demonstrated to be necessary in distraction of the mandible.

67
Q

A 25-year-old man who sustained a fracture of the maxillary alveolus involving the right central and lateral incisors is scheduled to undergo operative reduction and application of an arch bar in the emergency department for stabilization of the fracture. Adequate local anesthesia in this patient involves blockade of which of the following nerves?

(A) Greater palatine and anterior superior alveolar
(B) Greater palatine and buccal
(C) Nasopalatine and anterior superior alveolar
(D) Nasopalatine and buccal
(E) Infraorbital and middle superior alveolar

A

The correct response is Option C.

This 25-year-old man is to undergo operative reduction of a fracture of the maxillary alveolus involving the right central and lateral incisors, followed by application of an arch bar. To obtain a sensory blockade, the surgeon must anesthetize the nasopalatine nerve to block the palate (lingual surface) and the anterior superior alveolar nerve to block the teeth and alveolar mucosa (buccal surface). The nasopalatine nerve originates from the infraorbital nerve and passes through the incisive foramen to reach the anterior hard palate. It innervates the premaxilla at this point, then extends posteriorly to innervate the maxillary cuspids. The anterior superior alveolar nerve branches from the infraorbital nerve after it exits the infraorbital foramen and provides innervation to the maxillary incisors and canine teeth.

The greater palatine nerve emerges from the greater palatine foramen and provides sensation to the posterior portion of the hard palate.

The buccal nerve provides sensation to the buccal mucosa and lower gingiva.

The middle and posterior superior alveolar nerves are derived from the infraorbital nerve after it exits the pterygopalatine fossa. The middle superior alveolar nerve innervates the bicuspids, and the posterior superior alveolar nerve innervates the first, second, and third molars within the maxilla.

68
Q

In a 27-year-old man who has sustained bilateral parasymphyseal fractures, which of the following muscles exerts a distractive force on the anterior fracture segment?

(A) Geniohyoid
(B) Lateral pterygoid
(C) Masseter
(D) Medial pterygoid
(E) Posterior belly of the digastric muscle

A

The correct response is Option A.

Both anterior and posterior muscles exert forces on the mandible. The anterior muscles consist of the geniohyoid, genioglossus, mylohyoid, and digastric muscles. The muscles from this group exert primary distractive forces on the anterior fracture segment of a parasymphyseal fracture, displacing the fracture segment downward, posteriorly, and medially. The geniohyoid muscle originates from the mental spine of the inner anterior mandible and inserts on the hyoid bone. It acts to depress and retract the mandible.

The masseter, temporalis, and medial and lateral pterygoids comprise the posterior muscles. These muscles do not exert any force on the anterior segment of a parasymphyseal fracture.

The digastric muscle has an anterior and a posterior belly. The posterior belly originates on the medial aspect of the mastoid and courses forward and inferiorly as a tendon, passing through a fascial sling on the hyoid to transition into the anterior belly, which inserts into the digastric fossa of the mandible. The posterior belly of the digastric muscle primarily functions to elevate the hyoid and exhibits only a secondary effect on the anterior mandible. In contrast, the anterior belly of the digastric muscle exerts force on the anterior fracture segment.

69
Q

Simultaneous ipsilateral contractions of which of the following muscles produce the side-to-side grinding movements of the mandible?

(A) Masseter and lateral pterygoid
(B) Masseter and medial pterygoid
(C) Masseter and temporalis
(D) Medial and lateral pterygoid
(E) Medial pterygoid and temporalis

A

The correct response is Option D.

Simultaneous contractions of the medial and lateral pterygoid muscles largely produce the side-to-side grinding and chewing movements of the mandible. Both pairs of medial and lateral pterygoid muscles have two heads. The heads of the medial pterygoids originate from the medial surface of the lateral pterygoid plate and the tuberosity of the maxilla and insert into the medial surface of the mandibular angle and ramus. These muscles act to elevate the
mandible. The heads of the lateral pterygoids originate from the lateral surface of the lateral pterygoid plate and the infratemporal surface of the greater wing of the sphenoid and insert into the neck of the mandibular condyle and articular disk. These muscles act to protrude the mandible forward and open the mouth. When the ipsilateral medial and lateral pterygoid muscles work together, rotation occurs around the vertical axis of the contralateral condyle. Grinding and chewing movements occur when both sides alternate this action in rhythmic fashion. Simultaneous action of all four pterygoid muscles results in protrusion of the mandible.

The masseter muscle arises from the lower border and medial surface of the zygomatic arch and attaches to the lateral aspect of the mandibular ramus. It functions primarily to elevate the mandible to occlude the teeth. The temporalis muscle originates in the temporal fossa of the cranium and attaches to the coronoid process of the mandible. Although its primary function is elevation of the mandible, it can also retract the mandible because of the action of the posterior muscle fibers. Both the masseter and temporalis muscles contribute only minimally to the side-to-side grinding movements of the mandible.

70
Q

Which of the following nerves supplies sensory innervation to the buccal mucosa?

(A) Trigeminal (V) nerve
(B) Facial (VII) nerve
(C) Glossopharyngeal (IX) nerve
(D) Vagus (X) nerve
(E) Lingual nerve

A

The correct response is Option A.

The buccal branch of the trigeminal (V) nerve provides sensation to the buccal mucosa. It is important for the surgeon to know the anatomy of this nerve branch to plan and perform neurotized free flap reconstruction and reinnervation of the intraoral cavity.

The buccal branch of the facial (VII) nerve innervates the muscles surrounding the buccal mucosa.

The glossopharyngeal (IX) and vagus (X) nerves do not provide sensory innervation to the intraoral mucosa.

The lingual nerve provides sensation to a portion of the tongue.

71
Q

A 29-year-old man is involved in a fistfight and sustains bilateral fractures of the mandible in the region of the canine teeth. On examination, the fractures are vertically and horizontally unstable. The central segment of the mandible is most likely to be displaced in which of the following directions?

(A) Downward and anterior
(B) Downward and posterior
(C) Rotationally
(D) Upward and anterior
(E) Upward and posterior

A

The correct response is Option B.

This 29-year-old man has sustained bilateral fractures of the mandible that are located in the region of the canine teeth and are unstable both vertically and horizontally. Therefore, the central segment is most likely displaced downward and posteriorly due to the action of the anterior bellies of the digastric muscles and the geniohyoid and genioglossus muscles. In addition, the power of the mylohyoid muscle pulls the posterior fracture segments medially.

Rotational displacement of the central segment may occur if the fracture is stable either vertically or horizontally because the action of the muscles would then be opposed partially. Rotational displacement is characterized by hinging of the central segment on the upper or lower border of the mandible.

Upward and anterior displacement occurs only if the central fragment becomes detached from surrounding muscles, which is unlikely.

72
Q

Which of the following muscles of mastication pulls the mandible upward, medially, and forward?

(A) Anterior digastric
(B) Masseter
(C) Medial pterygoid
(D) Mylohyoid
(E) Temporalis

A

The correct response is Option C.

Knowledge of the muscles of the mandibular region and their functions in the movement of the mandible and displacement of fracture segments is crucial to accurately diagnosing and effectively treating facial fractures.

The medial pterygoid muscle exerts upward, medial, and forward traction on the mandible. This muscle arises inferiorly, laterally, and posteriorly from the medial surface of the pterygoid plate and inserts on the medial ramus and mandibular angle. Contraction of the medial pterygoid muscle pulls the mandible medially, elevating the lower jaw. A patient who sustains a fracture medial to the mandibular angle has displacement of the mandibular ramus medially and cephalad because of the forces of this muscle.

The anterior belly of the digastric muscle originates from the inside lower border of the symphysis and attaches to the lateral corner of the hyoid bone, while the posterior belly of the digastric extends between the hyoid bone and mastoid notch of the temporal bone. The digastric muscle is part of the suprahyoid musculature, which in a patient with a mandibular fracture would pull the anterior mandibular fragments posteroinferiorly.

The masseter muscle extends from the medial and lateral surfaces and lower border of the zygomatic arch to the anterolateral surface of the mandibular ramus. It exerts powerful forces of elevation on the mandible. In patients with zygomatic fractures, displacement of the zygoma is exacerbated by the chronic active traction of the masseter muscle. When performing open reduction of a zygomatic fracture, two-point wire fixation of the zygoma at the orbital rim will not diminish the forces of the masseter muscle and will instead create an axis around which the zygoma can rotate.

Because the bundles of the mylohyoid muscle join in the midline to form the muscular floor of the mouth, this muscle has only minimal influence on the movement of the mandible and hyoid bone. Instead, the mylohyoid functions to elevate the tongue.

The temporalis muscle elevates and retracts the mandible. In patients with subcondylar fractures, this muscle may elevate the mandibular ramus, resulting in loss of posterior facial height.

73
Q

According to the Angle classification, which of the following permanent teeth are used to determine the class of dental occlusion?

(A) Central incisors
(B) Lateral incisors
(C) First premolars
(D) Second premolars
(E) First molars

A

The correct response is Option E.

According to the Angle classification, the permanent first molars are used to determine the class of dental occlusion. Angle class I, or normal, occlusion is defined as the mesiobuccal cusp of the maxillary first molar lying in the buccal groove of the mandibular first molar. In Angle class II malocclusion, the mesiobuccal cusp of the maxillary first molar lies mesial (anterior) to the buccal groove of the mandibular first molar. Angle class III malocclusion is defined as the mesiobuccal cusp of the maxillary first molar lying distal (posterior) to the buccal groove of the mandibular first molar and being located instead in the buccal groove of the lower second molar.

The central incisors are used to determine overbite and overjet. Both overbite and overjet refer to the distance between the maxillary and mandibular incisors; overbite is measured vertically, and overjet is measured horizontally. In reverse overjet, which can occur in patients with Angle class III malocclusion, the maxillary central incisor lies distal (posterior) to the mandibular incisor.

The premolars and lateral incisors are less relevant than the molars and central incisors in the evaluation of occlusion.

74
Q

A 25-year-old woman is undergoing evaluation of microgenia. Cephalometric analysis shows marked bony deficiencies in both the anteroposterior and vertical dimensions. Occlusion is normal. Which of the following is the most appropriate management?

(A) Chin implantation
(B) Intraoral vertical osteotomy
(C) Sagittal split osteotomy
(D) Osseous genioplasty
(E) Distraction osteogenesis

A

The correct response is Option D.

In this patient who has microgenia with anteroposterior and vertical deficiencies of the chin, the most appropriate management is osseous genioplasty. Microgenia is one of several terms used to describe abnormalities of the chin. In this condition, the chin is small and bone is deficient in all three planes. Macrogenia describes a large chin; both macrogenia and microgenia can be associated with normal occlusion or mandibular prognathism. In patients with retrogenia, the chin is positioned posteriorly but is not necessarily small. Occlusion is normal in pure retrogenia. Patients with mandibular retrognathia have secondary retrogenia. Any of the above conditions can be associated with vertical abnormalities and chin asymmetry.

This patient has moderate-to-severe vertical microgenia, which frequently includes anteroposterior deficiency of the chin. This results in decreased height of the lower face, creating an imbalance between the midface and lower face. This abnormality can be corrected by performing osseous genioplasty, which will increase the projection of the mental symphysis within the sagittal plane while compensating for the vertical deficiency.

Chin implantation is recommended to increase the anteroposterior projection of the chin but does not correct vertical deficiency or excess.

Both intraoral vertical osteotomy and sagittal split osteotomy of the mandibular ramus will alter dental occlusion and are not indicated for correction of vertical or horizontal deficiencies, especially in a patient who has normal occlusion.

Distraction osteogenesis is indicated for treatment of skeletal deficiencies (ie, Pierre Robin sequence) but is an unnecessary, excessive option for correction of this deformity.

75
Q

A 31-year-old man sustains a fracture of the condylar neck of the mandible. On physical examination, the condylar fragment is displaced medially. This finding is most likely caused by the action of which of the following muscles?

(A) Buccinator
(B) Lateral pterygoid
(C) Masseter
(D) Medial pterygoid
(E) Temporalis

A

The correct response is Option B.

Fragments of fractures of the condylar neck are displaced by the action of the lateral pterygoid muscle. This muscle has two heads. The inferior head arises from the lateral pterygoid plate and inserts into the pterygoid fovea on the neck of the mandible; in patients with subcondylar fractures, the inferior head will pull the condylar fragment medially, resulting in the displacement seen in this patient. In contrast, the superior head of the lateral pterygoid arises from the greater wing of the sphenoid and inserts into the articular surface and disk of the temporomandibular joint.

The buccinator muscle is one of a group of muscles of facial expression. It is innervated by the buccal branches of the facial (VII) nerve and arises from the pterygomandibular raphae to insert into the orbicularis oris muscle and the mucosa and skin of the lips. It acts to flatten the cheek against the teeth. Because it does not arise within the condylar region, it does not exert any forces on the condyle or mandible.

The masseter, medial pterygoid, and temporalis muscles all insert below the fracture line within the condyle. Therefore, these muscles do not exert any pull on the condylar segment. The masseter muscle arises from the zygomatic arch and inserts into the lateral and inferior portion of the mandibular ramus. The medial pterygoid arises from the medial surface of the lateral pterygoid plate and inserts into the medial and inferior borders of the mandibular ramus. The temporalis muscle originates within the temporal fossa and inserts into the coronoid process of the mandible and the medial side of the mandibular ramus.

76
Q

A patient has an infection at the surgical site one week after undergoing open reduction and internal fixation of a fracture of the mandibular body using an inferior border reconstruction plate and a tension band. Occlusion is normal. The infection site is surgically drained; intraoperative exploration shows that the plates and screws are stable with no evidence of loosening.

Which of the following is the most appropriate management of the hardware?

(A) Maintenance of current stabilization without removal of the hardware
(B) Removal of all plates and immediate application of intermaxillary fixation
(C) Removal of all plates and immediate application of two miniplates
(D) Removal of all plates and immediate placement of an external fixator
(E) Removal of all plates and placement of new plates when the infection has subsided

A

The correct response is Option A.

Infections following open reduction and internal fixation of mandibular fractures typically result from failure of fixation devices, for example, loosening of the screws. In addition to operative drainage of the infection and antibiotic therapy, appropriate management in the majority of these situations includes removal and replacement of the hardware with intermaxillary or external fixation for stabilization of the fracture.

In this patient, the plates continue to provide stable fixation of the fracture. Because of this, the current stabilization should be maintained without removal of the hardware, and the patient should undergo operative drainage of the infection and administration of antibiotics.

77
Q

A 36-year-old woman has pain in the right side of the mandible after falling and striking her face on a sidewalk. Physical examination shows numbness of the right lower lip. The most likely cause of these findings is a fracture of which of the following regions of the mandible?

(A) Ascending ramus
(B) Body
(C) Condyle
(D) Coronoid process
(E) Symphysis

A

The correct response is Option B.

These findings are most consistent with a fracture of the body of the mandible. The mandibular angle and body are fractured most commonly. Fractures of the mandibular body are most likely to cause impingement or transection of the inferior alveolar nerve, resulting in numbness of the right lower lip. This nerve enters the mandible at a point proximal to the mandibular angle, travels through the angle and body to the mental foramen opposite the first bicuspid tooth, then emerges as the mental nerve, supplying sensation to the soft tissues of the lip and chin. Physical examination and radiographs are most likely to confirm the location of the fracture.

Fractures of the ascending ramus, condyle, coronoid process, and symphysis are less common than fractures of the mandibular angle and body and would not result in numbness of the lower lip.

78
Q

A 10-year-old boy has a laceration of the chin and pain in the jaw and ear after falling while ice skating. On examination, the maximal incisal opening is 10 mm, and the chin point is deviated to the left. There is an upward cant of the mandibular occlusion on the left with a right-sided lateral open bite.

These findings are most consistent with which of the following?

(A) Bilateral condylar fractures
(B) Bilateral temporomandibular joint dislocation
(C) Left-sided condylar fracture
(D) Left-sided mandibular body fracture
(E) Right-sided condylar fracture

A

The correct response is Option C.

The findings in this child are most consistent with a left-sided condylar fracture. It is necessary to exclude a diagnosis of condylar fracture in any child who sustains trauma to the chin. Indications for a diagnosis of condylar fracture include malocclusion, pain with range of motion of the temporomandibular joint, and preauricular pain. Lacerations of the external auditory canal may also be associated. Patients with unilateral condylar fractures exhibit loss of posterior ramus height unilaterally, resulting in premature contact of the maxillary and mandibular molars posteriorly and a contralateral lateral open bite. The mandibular occlusal plane will demonstrate an ipsilateral upward cant. The maximal incisal opening will be decreased, and the chin point and mandibular midline will be deviated ipsilaterally due to the unopposed action of the lateral pterygoid muscle on the contralateral side. Because this child has chin deviation and an upward cant on the left with a right-sided lateral open bite, a left-sided condylar fracture can be diagnosed.

A child with bilateral condylar fractures will have an anterior open bite resulting from premature contact of the mandibular and maxillary molars posteriorly. Ear pain and lacerations of the external auditory canal may also be present bilaterally.

Bilateral temporomandibular joint dislocation typically results in an open bite and severe limitation of jaw excursion, also known as “lock-jaw.”

Although left-sided mandibular body fractures can be associated with limited mouth opening, a contralateral open bite and an ipsilateral upward occlusal cant are not typical of this type of fracture.

As mentioned above, a right-sided condylar fracture would manifest as a left-sided lateral open bite with chin deviation and an upward occlusal cant on the right.

79
Q

A 24-year-old woman sustains facial injuries in a motor vehicle collision. On examination, there is tenderness in the preauricular region bilaterally, posterior facial height is decreased, and there is malocclusion with an anterior open bite. Panoramic radiographs show low subcondylar fractures of the mandible bilaterally. The mandibular condyles are seated within the glenoid fossa, and the proximal segment overrides the distal segment laterally.

Which of the following is the most appropriate management?

(A) Observation
(B) Intermaxillary fixation for two weeks followed by physical therapy
(C) Intermaxillary fixation for eight weeks followed by physical therapy
(D) Bilateral external fixation
(E) Open reduction and internal fixation

A

The correct response is Option E.

Because stable anatomic reduction of the fracture segments is crucial for management of this patient’s injuries, open reduction and internal fixation should be performed via a preauricular approach. Accurate reduction of a subcondylar fracture is rarely achieved with closed reduction alone. In addition, the absence of internal fixation will lead to fracture instability secondary to the forces of the masseter, temporalis, and medial and lateral pterygoid muscles, ultimately resulting in decreased posterior facial height and abnormal condylar mechanics caused by displacement of the condylar head. The patient will be at greater risk for malocclusion and development of degenerative osteoarthritis. Therefore, accurate open reduction with rigid internal fixation is advocated to avoid any potential complications. With this approach, normal posterior facial height will be restored, and the risk for abnormal joint mechanics will be minimized. Endoscopically-assisted fracture reduction, with rigid fixation, is a new technique that shows promise because it combines the advantages of the open approach (ie, anatomic reduction and early motion) while minimizing external scarring and the risk for facial nerve injury.

Observation alone is inadequate fracture management and will result in malunion, nonunion, and/or the development of pseudarthrosis.

Although a short course of intermaxillary fixation (two to three weeks) followed by graduated opening of the mandible has traditionally been implemented in the management of subcondylar fractures, it does not address fracture malalignment or its potential complications. Prolonged intermaxillary fixation (six weeks or more) is associated with an increased risk for temporomandibular joint stiffness and a subsequent decrease in interincisal opening.

80
Q

In pediatric patients, abnormalities in mandibular growth are most closely associated with fractures involving which of the following regions of the mandible?

(A) Angle
(B) Body
(C) Condyle
(D) Ramus
(E) Symphysis

A

The correct response is Option C.

In children, abnormalities in mandibular growth are most closely associated with fractures involving the mandibular condyle. More than one-third of all facial fractures in children involve the mandible. The pediatric condyle, which is the primary growth center of the mandible, is immature, highly vascular, and covered with a thin sheath of periosteum. Any compressive or traumatic forces may cause the condyle to burst, rather than fracture, resulting in fragmentation of bone, hemarthrosis, and increased osteogenic potential. Ankylosis and growth disturbances are potential long-term complications. According to one study, 47% of children who sustained condylar fractures subsequently developed temporomandibular joint dysfunction or abnormalities of facial growth.

In order to maintain the height of the mandibular ramus and adequate function of the temporomandibular joint, appropriate management should include closed reduction and immobilization for five to eight days, followed by initiation of physical therapy.

81
Q

A 38-year-old man has paresthesias of the lip and chin on the side of the fracture after rigid fixation is applied for management of a nondisplaced fracture of the mandibular angle. He has complete dentition with class I occlusion; sensation of the lip and chin was normal preoperatively.

The most likely cause of his current findings is injury to which of the following nerves?

(A) Buccal
(B) Facial
(C) Inferior alveolar
(D) Superior alveolar
(E) Lingual

A

The correct response is Option C.

This patient who has paresthesias of the lip and chin following the application of rigid fixation has most likely sustained an injury to the inferior alveolar nerve. Injury to this and other sensory nerves occurs frequently in patients with mandibular fractures. In patients with displaced fractures, the incidence of injury has been reported to range from 11% to 59%. Although most injuries involve neurapraxias that develop secondary to stretching or compression, traumatic sensory nerve damage is most likely to occur within the intrabony course of the inferior alveolar nerve. This patient’s injury most likely occurred during placement of a cortical screw into the mandibular canal, which is common. Other causes include bony fracture displacement and extraction of third molars.

Because the buccal, facial, and lingual nerves are not found within the bony canal of the mandible, they are not likely to be involved with fracture. The superior alveolar nerve provides sensory innervation to the maxilla and thus would not be affected by a mandibular fracture.

82
Q

Which of the following mechanisms of action of the mandible occurs during the terminal 4 cm to 5 cm of jaw opening?

(A) Rotation within the lower joint space
(B) Rotation within the upper joint space
(C) Translation and rotation within the lower joint space
(D) Translation within the lower joint space
(E) Translation within the upper joint space

A

The correct response is Option E.
Normal opening of the mandible results from the synchronized movements of muscles surrounding the joint space. The articular disk separates the joint space into upper and lower spaces. The combination of motions of the mandibular condyle generates the motion of the temporomandibular joint. At rest and during rotation, the mandibular condyle is located in the lower joint space. During translation, the condyle moves into the upper joint space.

Most patients have a maximal incisal opening of 4 cm to 5 cm. The initial 1 cm to 2 cm of jaw opening involves rotatory, or hinge, movements. Jaw opening at 2 cm to 3 cm is a combination of rotation and translation. The terminal 3 cm to 5 cm of jaw opening involves translatory movements only.

83
Q

Distraction osteogenesis of the mandible in an 8-year-old boy is optimally performed at a rate of how many millimeters daily?

(A) 0.1
(B) 0.5
(C) 1.0
(D) 2.0
(E) 5.0

A

The correct response is Option C.

In a patient who is undergoing distraction osteogenesis for bone lengthening to compensate for a deformity, the distraction zone forms a radial pattern, allowing for the formation of bone at different rates within the zone. Although experimentation with different rates of distraction has had positive results with rates from 0.5 mm to 2.0 mm daily, distraction at a rate of 1.0 mm has been shown to be optimal in most situations, including mandibular lengthening in an 8-year-old child. Some surgeons are performing distraction osteogenesis in infants at rates as high as 2.0 mm daily because of the greater osteogenic potential seen in infants, which allows for an acceleration of the process. However, this high rate is associated with delays in ossification, especially in areas of low metabolism, such as the diaphysis. Rates of 0.5 mm daily or less are associated with an increased risk for premature consolidation.

84
Q

A 21-year-old man sustains blunt trauma to the face while playing football. On examination, he has unilateral pain and facial swelling; he is unable to open his mouth. Radiographs show a nondisplaced coronoid fracture. Which of the following is the most appropriate initial step in management?

(A) Coronoidectomy
(B) Maxillomandibular fixation
(C) Endoscopic reduction and fixation
(D) Open reduction and rigid internal fixation
(E) Open reduction and wire fixation

A

The correct response is Option B.

In this 21-year-old man who has an isolated nondisplaced fracture of the coronoid process, the most appropriate management is short-term maxillomandibular fixation. The tendons of the temporalis muscle act as splints for the fracture fragments, allowing osseous union to occur spontaneously. Because ankylosis may develop between the coronoid process and the zygomatic arch if immobilization is prolonged, fixation should only be applied for one to two weeks. Following removal of fixation, physical therapy may be required to re-establish the normal vertical dimension of the face.

In rare patients who have displaced fracture fragments that obstruct normal mandibular motion, partial coronoidectomy may be performed. Endoscopic approaches to the temporomandibular joint are reserved for intra-articular injuries such as meniscal tears. Rigid or wire fixation is not required because of the expected osseous union associated with this type of fracture.

85
Q

An edentulous 65-year-old man sustains bilaterally displaced fractures of the mandibular body in a motor vehicle collision. Which of the following is most effective for determining the patient’s maxillomandibular relationship prior to the application of rigid fixation?

(A) Analysis with a face-bow
(B) CT scans of the face
(C) Custom-fabricated intraoral splints
(D) Erich arch bars
(E) Plain radiographs

A

The correct response is Option C.

Management of maxillomandibular fractures involves determination of the anatomic relationship of the maxilla and mandible to each other, as well as to the cranial base. After the orientation of the jaws has been established, the fracture segments can be exposed and rigid fixation can be applied. Human teeth are typically used to establish maxillomandibular orientation; however, this is not possible in the edentulous patient. Custom-fabricated intraoral splints or the patient’s own dentures can instead be rigidly fixed to the maxilla and mandible using wire or screws. The maxilla and mandible can then be brought into occlusion and fixed together.

A face-bow is used to determine the relationship of the maxilla and the midface to the cranial base; the mandible is not assessed. Plain radiographs and CT scans of the face are also helpful in determining the extent of the patient’s injuries and in planning surgery, but not in establishing skeletal orientation. Erich arch bars can only be used in patients with functional dental occlusion.